For Discussion: Can convection neutralize the effect of greenhouse gases?

Here’s an idea for commenters to sink their teeth into — Stephen Wilde postulates a mechanism where convection can neutralize the effect of changes in  greenhouse gases. The focus in David’s series of blog posts has been on radiation, but the troposphere is governed by convection.  The tropopause is the boundary where convection runs out of oomph, and above the tropopause, in the stratosphere, radiation rules supreme. Airliners like to fly at the bottom of the stratosphere (“thirty thousand feet”), just above the convection and water vapor in the troposphere, because it’s calm. But sometimes turbulence from the troposphere punches up into the stratosphere, so think of Fig 1 below if you are asked to suddenly sit in your seat and fasten your seat belt.

All the focus on radiation imbalances tends to ignore the powerful effects of gravity and convection. To get a sense of how important gravity is, ponder that a mere 3km above the surface the pressure is 300hpa lower, but gravity stops  the surface air from rushing up and equalizing that. Imagine if there were two sites on the ground where pressure was, say,  1000hpa and 700hpa, and they were only three kilometers apart, we’d have some kind of monster Cat-10 cyclone on our hands.  (If gravity ever “stopped”, we get some sense of just how fast the air would leave the planet.)

I have this vision of the tropopause as being flat (as it is in so many diagrams) but Stephen Wilde points out that it’s a lumpy roiling creature. Sometimes it’s a lot further or closer to the surface than the “average” and in that variation lies a kind of loophole. Air is constantly rising and falling in a tricky “adiabatic” way — where blobs of air (officially called “parcels”) rise and fall without mixing with the surrounding air. A blob heats up over a desert, say, and rises, expanding to the the top of the tropopause  (or overshooting it). Blobs of cold air can also sink and reverse that process, but Wilde points out that it isn’t symmetrical. The blobs can overshoot at the top, then slide and mix with the stratosphere in a non-adiabatic way, until they reach a lower altitude before turning into a “parcel” and sinking back to the surface. That gap where the air slides from a high spot in the bumpy tropopause to a low point is important.

Stratosphere, Tropopause, intrusions

The troposphere is constantly churning – low pressure systems work like vacuum cleaners drawing warm air up from the surface to the tropopause. High pressure cells do the opposite, drawing in cold air from the tropopause and pulling it back down to the surface. Generally the high’s are broad and slow moving, and the lows are smaller, faster and there are more of them.

There is constant interplay between convection and radiation, with convection attempting to follow the lapse rate but radiation distorting it.

The “lapse rate” is how quickly it cools as one ascends — on average about 6.5 C per km of vertical ascent. If the air were perfectly dry it would be nearer 10 C per km (the dry adiabatic lapse rate), and if it were as wet as could be then it would drop to around 5 C per km (the moist or saturated adiabatic lapse rate).

Stephen Wilde, who has a long and avid interest in meteorology, explains an idea he has on interplay between radiation, convection and lapse rates. Though it is not quantified, it is worth a discussion. See where that leads…

— Jo

—————————————-

Neutralizing Radiative Imbalances Within Convecting Atmospheres

Stephen Wilde, 3 October 2015

This article sets out a simple mechanism whereby planetary atmospheres can be rendered thermally stable over time despite huge variations in the atmospheric content of radiatively active molecules such as greenhouse gases, material released by volcanic outbreaks of a vast size, and material vaporized in large asteroid or meteor strikes.

It is established science that convective adjustments can stabilise or neutralise radiative imbalances:

“Radiative equilibrium  profile could be unstable; convection restores it to stability (or neutrality)”

and:

Note that the hydrostatic equation depicts the vertical balance of force for a piece of fluid at rest. The balance is between the upward pressure gradient force and downward gravitational force The hydrostatic equation is the vertical component of the momentum equation (Newton’s equation of motion) for the fluid parcel when the forces are in  prefect balance and the net acceleration = 0.”

Readers could study that lecture since it explains the concept of hydrostatic balance within atmospheres.

It appears that those climate scientists who apply the radiative gases theory of climate change have overlooked the means by which convection neutralizes radiative imbalances.

Let’s start by looking closely at the tropopause, which is known to undulate upwards above rising air masses and downwards above falling air masses. The height of the tropopause is set by ozone in the stratosphere reacting directly with incoming solar energy so as to create a temperature inversion that blocks further upward convection.

The situation at the tropopause can become very complex:

Stratosphere, Tropopause, intrusions

Source: http://www.acom.ucar.edu/start/

 

The above picture focuses on frontal zone distortions between low pressure (ascent)) and high pressure (descent) across the latitudes between equator and poles but our interest here is on the interaction between the high and low pressure centers themselves at any latitude or at the same latitude so we can simplify the graphics thus:

 

 

pp

Figure 1

The first diagram shows, by way of a broken green, line an undulation in tropopause height between rising and falling columns of air.  If one then looks at the green arrows in the cloud to the left of the picture, the lateral flow corresponds with the high level winds in the simplified second diagram and in the absence of the frontal zone distortion one should extend the flow downwards beneath the distorted tropopause to the top of the adjoining subsidence column, which is actually situated to the right of the first diagram. That scenario applies between all high and low pressure cells even if they adjoin one another at the same latitude.


Taking an atmosphere in hydrostatic balance (as they all must be), the raised areas are, on average, over time, equal in volume to the lowered areas. If that were not the case then the atmosphere would constantly expand or constantly contract.

Cold air at the top of ascending columns is forced to one side by warmer air continuing to flow up from beneath but it is blocked by the warmer stratospheric air above and so flows laterally and downward to the top of descending columns, following the undulating slope of the tropopause.

Whilst flowing across the area between the two columns that cold air does not warm up by compression despite falling in height because it remains in contact with stratospheric air with which it can freely exchange energy by conduction and mixing.

Falling air in the descending column will not start to warm by compression until parcels of falling air become detached from the tropopause.

At that point, energy exchange with stratospheric air ceases and a process known as adiabatic descent begins. In meteorology an adiabatic process is one whereby no heat passes between the vertically moving parcel and the surroundings.

For those who wish to go deeper into that aspect this is a useful source: PSU Courseware

 

“As an air parcel rises, it moves into an environment of increasingly lower pressure (remember that pressure decreases with increasing altitude). In order to equalize the pressure difference between the the rising parcel and its new environment, air molecules inside the higher-pressure air parcel push out the sides of the parcel. That requires molecules to do work, which results in a loss of kinetic energy. With kinetic energy expended by air molecules to push out the sides of the expanding parcel, the temperature of the air inside the parcel decreases (recall that temperature corresponds to the average kinetic energy of molecules). The energy spent by molecules to push out the sides of the parcel amounts to a flat rate of 5.5 degrees Fahrenheit per 1000 feet of ascent (10 degrees Celsius per kilometer).”

The process is reversed in the descent phase.

The important point to note is that adiabatic descent involves a falling parcel of air contracting and heating despite there being no heat drawn by it from the surroundings. A rising or falling parcel in a pure adiabatic process will retain its temperature differential with its surroundings throughout the ascent or descent. No process is perfectly adiabatic so there will always be some diabatic energy exchange in or out of the moving parcel but here we are only concerned with the adiabatic portion. Any work done against surrounding molecules would be diabatic and not adiabatic.

Once the convective loop is established the air molecules in the direction of travel move away without providing any resistance

The parcel of air changes its own temperature and pressure at the same rate as the temperature and pressure of the surroundings changes. It follows that air rising and expanding into a region of lower pressure need do no work on the surrounding molecules because it simply expands into the additional space made available by the reducing density gradient..

Similarly, a parcel falling into a region of higher pressure simply contracts to fit into the reduced space made available by the increasing density gradient.

Now, bear in mind that convective uplift in the rising column had previously pushed tropopause height down above the descending column by the same volume as it pushed tropopause height up above the ascending column.

Recall that the air flowing across laterally and downward to the top of the descending column did not warm up adiabatically due to its contact with the stratosphere above.

What we then see is air at the top of the descending column colder than it ‘should’ be for its position along the dry adiabatic lapse rate slope AND a reduced distance from the surface so that it cannot warm up as much as it otherwise would have warmed up during its descent to the surface.

It follows that as a direct result of the tropopause having been raised above the rising column there is less warming by compression than there would otherwise have been at the base of the descending column.

The next point to consider is that the rising column only developed in the first place due to uneven surface heating which causes density variations in the horizontal plane. Lighter, warmer air will always rise above heavier, colder air within an atmosphere suspended off a surface against gravity in hydrostatic equilibrium. The declining density gradient with height (caused by gravity) permits lighter, warmer air to rise further away from the surface than can colder, heavier air at the same upward pressure gradient force.

Greenhouse gases are not required in order to cause uneven surface heating. Such unevenness at the surface is the inevitable consequence of the physical characteristics of a rough surfaced, rotating sphere illuminated by a point source of light such as a sun.

Convective overturning is therefore inevitable even with no radiatively active material in an atmosphere at all. Uneven conduction to the atmosphere from the illuminated surface is all that is needed combined with the declining density gradient with height.

So, we are now ready to pull all that together in order to show how a planetary atmosphere uses convective overturning to neutralise the effect of radiatively active materials of any type so that hydrostatic balance can be maintained whatever fate throws at it.

We will start with a theoretical radiatively inert atmosphere which will still have undulations at the tropopause due to uneven surface heating below but they will be minimal and so for all intents and purposes the rising and falling columns will both follow the dry adiabatic lapse rate as they move up and down:

 

pp

Figure 2

 Now we will turn to a non-condensing radiatively active material such as CO2 but the same principle applies to all other radiatively active materials such as various other gases and particulates.

pp

Figure 3: DALR = dry adiabatic lapse rate, about 10 deg C per km

.

 

The most important feature is the vertical displacement without a change in temperature (shown in green) because that is what changes the distance along the lapse rate slope which is available to allow adiabatic compression at the dry adiabatic lapse rate in descent.

The existence of such vertical displacements around convective columns is shown here:

Tropopause, Stratosphere, Gaps

Source: https://www2.acom.ucar.edu/news/cloud-tops-and-tropopause

Although the above picture shows the most dramatic scenario between the cloud tops at the equator and those at higher latitudes the truth is that the pattern is exactly the same but to a lesser degree around all convective columns at whatever latitude and whether clouds are present or not.

Taking the process step by step:

i) In an ascending column the CO2 at lower levels is a net absorber of radiation from the ground so that the slope of the lapse rate is reduced in an ascending column which slows down the rate of convection from the surface. The steeper the lapse rate slope the faster is convection so reducing the slope reduces the speed of convection. Reducing upward convection allows the surface to warm beneath the rising column to a higher temperature than it otherwise would have done.

Note that the position is exactly reversed in the descending column. Whereas the CO2 has reduced the rate of uplift in the ascending column (which warms the surface) CO2 at lower levels reduces the rate of descent in the descending column which reduces surface temperature because less warmth is then being generated via compression of descending air.

At higher levels CO2 becomes a net radiator of energy to space so that the slope of the lapse rate increases again. The cooling higher up reinvigorates convection from below and since the entire ascending column contains more energy than it otherwise would have done the tropopause is raised more above the rising column than would otherwise have been the case.

Kinetic energy at the surface acting via conduction and convection supplies the upward pressure gradient force which offsets the downward force of gravity in order to constantly hold the mass of an atmosphere off the surface in hydrostatic balance.

In order to maintain hydrostatic balance the necessary kinetic energy cannot be radiated away to space hence that ‘additional’ energy must be held at the surface over and above that which is required (by the surface and atmosphere combined) solely to radiate enough energy to space to match energy being received from space. That additional kinetic energy at the surface is the true greenhouse effect and it is mass induced rather than GHG induced.

ii) We see an increased tropopause height above the rising column but, as described above, that induces a decreased height above the descending column so that the surface below the descending column warms up less during adiabatic descent than would otherwise have been the case. Thus the net thermal effect at the surface is zero.

iii) Temperature differentials at the surface between the base of rising columns and the base of falling columns will be enhanced but the average surface temperature remains unchanged. Surface winds increase to deal with the enhanced differentials across discrete regions at the surface.

iv) An AGW proponent might ignore the lack of any net surface temperature rise and instead seize on the idea that CO2 might make a large enough difference to increase storminess at the surface. To resolve that aspect one need only consider that convective overturning involves the entire mass of the atmosphere reacting to huge variations induced by solar and oceanic processes. Any contribution to average surface wind speed from CO2’s radiative capability relative to the overturning of atmospheric mass induced by conduction and convection would be magnitudes less and impossible to measure.

We should now look at water vapor as an example of a condensing radiatively active component of the atmosphere.

The scenario is similar:

pp

Figure 4

The difference lies in the absence of water vapor in the descending column which then warms at a different rate to the cooling in ascent. Additionally, the distortions of the lapse rate in ascent are greater than for CO2 because water vapor is lighter than air and contains more energy in latent form which heats the air around it when condensation occurs during uplift.

The above application of  known meteorological principles appears not to have been considered by the climate establishment.

To summarize, lapse rate distortions caused by GHGs always slow down the rate of convection at lower levels. In the ascent column that slows surface cooling via decompression for a warming effect and in the descent column that slows surface warming via compression for a cooling effect. To maintain hydrostatic balance the two effects must be equal and opposite.

My thanks to David Evans for creating the above diagrams from my hand drawn originals, and to Joanne Nova for tracing some source material that helped to substantiate the above narrative.

8.7 out of 10 based on 53 ratings

208 comments to For Discussion: Can convection neutralize the effect of greenhouse gases?

  • #
    AndyG55

    YES 🙂

    I have agreed with Steven on this for quite a long time.

    He just has the gift of the “telling” 🙂

    Well done, SW.

    223

    • #
      Truthseeker

      The discussion can get started once we stop using the term “Greenhouse Gas”.

      A real greenhouse works by blocking convection, not by blocking radiation.

      No free flowing gas blocks convection.

      Therefore no gas acts like a greenhouse.

      The term “greenhouse gas” is wrong and prejudicial (we know greenhouses are warmer than their surroundings – which is why we use them – and that makes the term prejudicial).

      Stop using it.

      90

      • #
        Peter C

        Ok,

        Radiative gases from now on!

        80

      • #

        Truthseeker,

        Descending, warming air reduces convection from below and thus can be said to act like a greenhouse roof.

        Furthermore, it dissipates clouds so as to create transparence just like a greenhouse roof.

        Thus there is a greenhouse effect but it is mass induced and not radiation induced.

        1410

        • #

          Stephen, your first diagram makes sense with regard to convection but it is rendered doubtful by the Chemistry which you do not mention in the article.
          The first chemical is formaldhyde is is quite difficult to make requiring catalysts and elevated temperature. It is possible there are parts per billion in the atmosphere but is just as likely that any measured amount comes from contamination in the measuring equipment. The second chemical in the diagram is formic acid something ants produce. It can be produced from methanol by oxidation of methanol again with catalysts at temperatures higher than those in the atmosphere.
          Methane CH4 generally does not oxidise in the atmosphere. It has a residence time around 10 years. It is slightly soluble in water, it can be taken up by some plants and some bacteria in oceans, in soils and on vegetation. CH4 can be oxidised by ozone (O3) which is produced by lightning, welding and from the decomposition of NOx Ozone also exists high in the stratosphere but because of low partial pressure of both ozone and CH4 in this part of the atmosphere the result is not significant.
          Please do not spoil your argument with junk from people who have no idea of the chemical engineering subject -reaction kinetics.

          10

          • #

            cementafriend,

            Surely those chemical reactions are pretty minimal and their thermal effects would simply involve an adjustment of lapse rates and convection too?

            I am giving the big picture, not covering every detail.

            84

            • #

              Yes the chemical reactions are insignificant but why put them in when they are wrong. I repeat- why spoil what maybe a good argument with junk. The movement of jet streams, the movement of magnetic poles and the changes of magnetic fields all could play an important part in short and long term cycles. The ozone layer exists and the distributions of concentrations of ozone alters possibly in line with some cyclical changes.
              The possible only affect of chemical changes could be large volcanic eruptions which results in emission of large volumes of HCl, Cl2 and CH4 high into the stratosphere where these may affect the ozone layer.

              00

              • #

                The height of the tropopause is set by the balance of ozone creation / destruction within the stratosphere which appears to depend on solar spectral changes. I can cannot ignore that.

                109

              • #

                Stephen Wilde October 26, 2015 at 5:56 am

                “The height of the tropopause is set by the balance of ozone creation / destruction within the stratosphere which appears to depend on solar spectral changes. I can cannot ignore that.”

                Why such a claim? Where is there any evidence, not fantasy that composition of the stratosphere affects tropopause height in any way? 🙁 What is your proposed method for setting such height? More from the toilet? 🙁

                113

        • #

          Stephen Wilde October 24, 2015 at 6:17 pm

          “Descending, warming air reduces convection from below and thus can be said to act like a greenhouse roof.”

          No descending air-mass/volume can warm (add heat or increase temperature) of anything including itself. As both density and temperature increases with decreasing altitude the descending absorb heat from the incoming higher temperature molecules There is no reduction in convection.

          “Furthermore, it dissipates clouds so as to create transparence just like a greenhouse roof.”

          What is the ‘it’ that dissipates clouds?

          “Thus there is a greenhouse effect but it is mass induced and not radiation induced.” The term is but another fantasy by899999999999999999/. your meteorology CCC group. Its only possible meaning is deception and scam. Yet you emphatically promote such scam. Is that not illegal barrister?

          115

          • #
            wert

            English is hard. In some cases, it is a horrible language. It makes me feel very dense, as a simple sentence can’t be parsed in my mind to completion. What part of warm you didn’t understand!

            One of the most astonishing features is that the verb ‘to warm’ can be used in a context where is means something is becoming warmer. And sometimes it means something is making something else warmer. (I just checked this from a dictionary)

            This distinction is, I believe quite important, yet many languages allow letting the reader / listener to infer the meaning in many many cases.

            So when gases are warming, are they becoming warmer or are they making something else warmer? I constantly have to guess what is being meant, I fear I get it wrong, and I fear Will is getting it sometimes also wrong (or just at least very differently what I got).

            “Descending, warming air reduces convection”

            I think this means air just becomes warmer?

            “No descending air-mass/volume can warm (add heat or increase temperature)”

            And this is the other warm, ‘to make something become warmer’. So a different thing.

            An insightful comment. thanks. – Jo

            00

            • #

              SW(“Descending, warming air reduces convection”)

              ‘I think this means air just becomes warmer?’

              WJ(“No descending air-mass/volume can warm (add heat or increase temperature)”)

              ‘And this is the other warm, ‘to make something become warmer’. So a different thing.’

              Even in English, In technical discussion ambiguity is to be avoided as a courtesy. Words like warm, cool, and the verb form of heat, need be expressed with the intent, unless the intent is to deceive, lawyer speak! My ‘No descending air-mass/volume can warm (add heat or increase temperature)of anything including itself.’ Can be improved to:
              Any descending atmospheric mass/volume,acting as operator, cannot warm (add heat or increase temperature) of anything including itself. Nor can it reduces convection from below! Sorry for any confusion!

              012

            • #
              David Cosserat

              wert,

              You have touched on a very important topic which plagues all of us who take part in these discussions.

              Most postings about the thermodynamics of the atmosphere and the role of GHGs are concerned with ‘steady state’ conditions during which the (long term) temperature stays constant. It is true that we often discuss a temperature change that is postulated would occur if the atmosphere’s CO2 is doubled. But that is a change from one steady state to another.

              I think that the answer when discussing steady state systems is to always employ language that doesn’t use the words ‘heating’ or ‘warming’ or ‘cooling’ at all – unless we are quite clear that the process we are discussing is actually causing a temperature change.

              We have to keep sight of the fact that the key property isn’t temperature – it is energy. But this word itself is also often used ambiguously. For example, some people have difficulty distinguishing between the energy flowing through a system and the energy contained within a system. This leads them to fatuous conclusions such as that the surface of Venus is incredibly hot because of the radiative properties of its almost 100% CO2 atmosphere. They don’t understand that a body can be incredibly hot at a constant temperature (can contain a lot of energy) yet have very little energy flowing through it. In the case of Venus its surface is very well insulated by its very dense atmosphere which allows little incoming energy from the Sun to reach its surface; and impedes equally effectively the flow of outgoing energy to space. By conflating energy content with energy flow, they assume that the small energy flow to the Venus surface must be ‘amplified’ in some mysterious way by the CO2. In fact it is not amplified at all.

              10

        • #
          Truthseeker

          Stephen, thank you for proving my point. No particular gas acts like a greenhouse. All gases have the same effect according to their mass which means classifying some gasses as “Greenhouse Gasses” is entirely wrong.

          At the ground where actual greenhouses exist, the non-enclosed areas act as you have described. The enclosed areas (greenhouses) do not because of the physical barrier to convection. So saying that the whole atmosphere is one huge “greenhouse” is completely wrong because it is the same as those non-enclosed areas back at surface level where the real greenhouses actually exist.

          The movement of air does change the rate of convection but that is just the normal variations of a non-linear chaotic system and is not a barrier that a real greenhouse is.

          39

          • #

            Please remember that the first reason for ‘atmosphere’ and convection is to supply FRESH from the poles, and dispatch STINKY at the equator to way out there! This heat transfer thingy is ’cause ‘engineer, forgot ’bout other things happn! 😉

            08

  • #
    Peter C

    For Discussion: Can convection neutralize the effect of greenhouse gases?

    Reed Coray has written a paper demonstrating that even a steel greenhouse will cool the surface of the inner sphere if it is connected by conducting threads (think convection). Hopefully he will post it here.

    20

    • #
      Reed Coray

      Peter,

      I believe I have already posted a description of my paper on this blog

      [see http://joannenova.com.au/2015/03/weekend-unleaded-2/#comments%5D

      and given a URL where the paper can be read in its entirety

      [see https://www.dropbox.com/s/ejn20s9glptl2qv/solid-sphere-shell_temperatures_with_conduction_heat_current_02PDF.pdf?dl=0%5D.

      70

    • #
      Reed Coray

      Peter,

      You wrote: “Reed Coray has written a paper demonstrating that even a steel greenhouse will cool [emphasis mine] the surface of the inner sphere if it is connected by conducting threads.Will cool is too strong because to me it implies cooling always occurs. Thermally conducting rods have the potential to cool the inner sphere, but depending on the number/properties of the rods the temperature of the inner sphere of a steel greenhouse may still rise relative to the inner sphere temperature in the absence of the steel greenhouse. It’s a minor point, but I prefer the statement: “Reed Coray has written a paper demonstrating that even a steel greenhouse may (or “can” or “has the potential to” cool the surface of the inner sphere if it is connected by conducting threads.

      20

      • #
        David Cosserat

        Reed,

        I agree with your correction. Without the conductive rods in your thought experiment, the core will always be warmer than the shell which is in turn will be warmer than its surroundings. Otherwise energy wouldn’t flow from the powered core to the shell or from the shell to the environment.

        All that installing the conductive rods between core and shell will do is to increase the transfer of energy from core to shell but will NEVER, EVER make the shell warmer than the core.

        There is no mystery at all about how the steel greenhouse thought experiment works (with or without the addition of your rods). It is all straightforward college level thermodynamics. The fuss over Eschenbach’s original articles was because some people did’t properly understand the math and assumed that radiation from the inner surface of the shell was in some mysterious way ‘heating the core’. Actually all it is doing is exerting a radiative potential towards the core in opposition to the radiative potential exerted by the core towards the shell. Because the core is hotter than the shell, its radiative potential wins out over the shell’s. Only the difference between the potentials constitutes a real energy flow from core to shell, in full accordance with the second law of thermodynamics.

        11

  • #
    pat

    O/T but a MUST-READ. have posted excerpts on jo’s previous thread:

    24 Oct: Australian: Graham Lloyd: Political potholes on the road to Paris climate change talks
    http://www.theaustralian.com.au/news/inquirer/political-potholes-on-the-road-to-paris-climate-change-talks/story-e6frg6z6-1227580481665

    20

  • #
    chrism

    yes, this is good to conceptualise, but very hard to quantify, and perhaps even harder to measure,
    in addition to the vertical mixing effect, i wonder if the boundary vertically is tied to the thermal total energy below –
    in other words the boundary shifts up as energy accumulates in the system,
    also this is characterised in the tropical space and as latitude varies away from the equator,
    the convection effects are in increasingly dry air,
    and once again as energy accumulates in the system this wet/dry latitudinal change may be pushed further from the equator,
    and once again easy to conceptualise, very hard to theorise about but possibly easier to measure

    30

    • #
      Bob Fernley-Jones

      @ Schism,
      “yes, this is good to conceptualise, but very hard to quantify, and perhaps even harder to measure…”

      Yes, absolutely, and the very interesting concepts are a tad variable around the planet.
      No problem; all we need is a new megabucks computer and lots of highly funded programmers to code it.
      Personally, I feel that simpler analyses should be inherently less controversial, and that unless controversy can be minimised, (not to deny that complex advanced hypotheses can be very interesting), then defeat of the CO2 nonsense as cherished by Turnbull, Hunt, and hordes of other victims of faith is rather unlikely.

      60

    • #
      Greg Goodman

      I agree. It’s a good description in general but the hand-waving arguments could be different when quantified.

      I repeat: I generally agree but ….

      One of the main reasons warm air is less dense is not just temperature but water vapour content. Most of what is being considered it over ocean and warm air over oceans, esp. tropical is wetter.

      The other key element of the argument is the cooling in following the tropopause. Contact ( conduction ) and mixing with the stratosphere is cited but no mention of radiative cooling.

      I would have thought that radiative cooling was a major factor. If not this needs to be mentioned and explained.

      Since downward radiating IR can only affect a few microns of surface water , it seem obvious to me that its primary effect will be increased evaporation. NOT surface warming.

      Therefore it will the effect of surface air being less dense due to WV content rather than its being warmer this will drive the increase in convective cooling.

      Almost all downward IR will be converted directly into increased convection without the need for ( even minimal ) surface warming.

      “Efficacy” ( TM Hansen et al ) of GHG to warm surface is negligible. AGW is minimal..

      If David’s pipes do not include convective cooling they should do.

      20

      • #
        David Cosserat

        Greg,

        You say: If David’s pipes do not include convective cooling they should do.

        You are making the error of assuming that David Evans’ pipes are some sort of funnels moving energy up the atmospheric column. In fact they are ‘exit sluices’ whose job is to radiate away to space energy that they draw from the atmosphere’s fund of kinetic energy (sensible heat).

        The aggregate rate at which they perform this energy transfer function, exactly balances the rate at which the earth’ atmosphere absorbs energy from the Sun, which occurs as follows:

        (1) Incoming radiation that is directly absorbed by the atmosphere, contributes about 39% to the fund.

        (2) Evapo-transpiration from the surface which transfers latent heat to the atmosphere where it is converted to sensible heat during precipitation, contributes about 40% to the fund.

        (3) Radiation from the surface which is absorbed in the atmosphere as sensible heat, contributes 12% to the fund.

        (4) Convection from the surface as sensible heat, contributes about 9% to the fund.

        If there were no way of removing energy from the atmosphere, its temperature profile would not be stable. David’s ‘exit sluices’ perform this vital function, radiating energy from the atmosphere’s fund of sensible heat away to space at an aggregate rate that exactly matches the aggregate rate at which the earth system absorbs energy from the Sun.

        00

  • #
  • #
    sir boabtree

    My science schooling finished with year 10 science in 1972 but the way that is explained even I can see what he is talking about and why it would work that way.

    90

  • #
    KinkyKeith

    Any good scientific story or model starts with the diagrams.

    These look good.

    Now to the reading.

    🙂 KK

    41

  • #
    bobl

    Stephen,
    As you know I am struggling with the idea that the lapse rate can remain constant if radiation is increased, so I must agree that convective overturning must play its part. I struggle to see how global warming caused by an imbalance of just a fraction of a Watt can’t be freely dissipated. As I have pointed out many times, it’s an LED Christmas fairy light (half a watt) for each column of air of 1m x 1m x 10-30km or 10,000-30,000 cubic meters of air free to convect.

    But
    I am having a hard time with the vertical displacement, (The green arrow) the parcel of air has a particular (average) kinetic energy and Potential energy, if the parcel falls then the change in PE must go somewhere either the air will heat up (KE increase) or the energy must be exchanged (it can’t just disappear). Where does it go.

    Secondly in the moist diagram (Fig 4) on the way up, as the saturated parcel of air reaches zero deg C, there should be a range where the chart goes vertically to account for the enthalpy of solidification? Which is missing in the downward parcel because the ice is left behind as cloud in the ascending column of air. I’d expect an almost vertical segment in the ascending air side as the air remains at a relatively constant zero degrees while the water condenses out and solidifies.

    20

    • #

      bobl October 24, 2015 at 2:26 pm · Reply

      “Stephen,
      As you know I am struggling with the idea that the lapse rate can remain constant if radiation is increased, so I must agree that convective overturning must play its part.”

      Something is maintaining the lapse rate. Convection and latent heat can only decrease the 9.8c/km one. Does the same gravity the establishes the lapse rate also maintain such. The Tropospheric column re-thermalizes during Solar eclipse at a rate faster than convection can move that much atmospheric mass around. Has anyone even bothered to check? Does the CCC just claim to know?

      “I am having a hard time with the vertical displacement, (The green arrow) the parcel of air has a particular (average) kinetic energy and Potential energy, if the parcel falls then the change in PE must go somewhere either the air will heat up (KE increase) or the energy must be exchanged (it can’t just disappear). Where does it go.”

      The atmospheric energy does not necessarily change with altitude although temperature does. It is all tied up in the lapse rate and the cause of lapse rate; the constant atmospheric isentropic exponent with altitude. If heat is added to a gas at constant volume, both temperature and pressure must increase. This cannot be true at a constant or decreasing pressure. This atmosphere has an arbitrary volume for a given mass, at any altitude. As a volume is heated, anywhere in the atmosphere molecules leave that volume. The reverse happens when that volume cools. Then its density increases. At any point in the atmosphere both molecular random velocity and changes in momentum (direction) determine energy and temperature.
      As density increases the rate of change in momentum (banging into) increases the so called sensible heat; but since every gas has at least two, which one? None of this has anything to do with Newtonian kinetic or potential energy. Such from the likes of Steven are but part of the CAGW scam.
      Nowhere in the atmosphere need energy be conserved. The secondary function of an atmosphere is to dispatch excess entropy to space by any means available. It has and still is doing just that very well in spite of CCC nonsense.
      All the best! -will-

      612

    • #

      bobl October 24, 2015 at 2:26 pm

      “Secondly in the moist diagram (Fig 4) on the way up, as the saturated parcel of air reaches zero deg C, there should be a range where the chart goes vertically to account for the enthalpy of solidification? Which is missing in the downward parcel because the ice is left behind as cloud in the ascending column of air. I’d expect an almost vertical segment in the ascending air side as the air remains at a relatively constant zero degrees while the water condenses out and solidifies.”

      Now this part is an example of good observation and thinking!
      Thank you Bob! Atmospheric H2O liquid does persist down to -43 Celsius 230K.
      As WV condenses in the atmosphere, as it releases latent heat it must increase density, but no one knows by how much or in what shape. It certainly appears that most H2O colloids from WV remain airborne. No more than 10% of atmospheric column water is involved in precipitation or surface evaporation.
      HOW do the CCC meteorologists explain this? How much of these colloids are continually being evaporated sunside atmosphere. Then dispersed to all other locations via convection and rotation. Then discarding all that latent heat to space upon condensation back to airborne condensate. Where is that in the CCC energy budget?
      All the best! -will-

      612

      • #
        gnomish

        “As WV condenses in the atmosphere, as it releases latent heat it must increase density, but no one knows by how much or in what shape”

        one liter of H2O gas at stp, upon condensing, fills a teaspoon.
        the difference in volume between a liter and a teaspoon is far more than your TPV = TPV equation considers.

        water gas is also the lightest gas in our atmosphere (discounting H and He which are trace elements) so it rises whether it’s warmer than surrounding or not. convection is not required.

        10

        • #
          Bob Fernley-Jones

          @ gnomish,
          That seems ok intuitively but I don’t think so in practice. If you connect two containers holding different gasses with a tube, eventually they will mix equally each side under what is known as chemical dispersion

          10

          • #
            gnomish

            eventually, when all is in equilibrium
            until then hadley gets the first shot at stirring the blend.
            and fog, clouds, rain, mist, and lakes, rivers, glaciers conspire against it.
            ocean is large and severely equatorial
            hard to equlibriate on a rotisserie.
            and entropy is very sneaky.

            20

      • #

        I have a nice series of photos showing just that near Nebraska one day. I was driving cross country, and took a series each few hours. Clear at sunrise, vapor rises, makes very low cloud, midday, that evaporates and rises, makes mid level cloud. Repeat a couple of times. By evening, high cloud… Then clear the next morning…

        It is water, clouds, and ice thar matter.

        20

    • #

      bobl,

      “I am having a hard time with the vertical displacement, (The green arrow) the parcel of air has a particular (average) kinetic energy and Potential energy, if the parcel falls then the change in PE must go somewhere either the air will heat up (KE increase) or the energy must be exchanged (it can’t just disappear). Where does it go.”

      I mention that the temperature of the falling air is kept stable despite the decrease in PE (and increase in KE) because it remains in contact with the falling slope of the tropopause and so can freely mix with the stratospheric air above in a diabatic energy exchange. In effect.the extra heat (from radiative gases) in the ascending column leaks up into the stratosphere whilst the cold air slides laterally and downward.

      “Secondly in the moist diagram (Fig 4) on the way up, as the saturated parcel of air reaches zero deg C, there should be a range where the chart goes vertically to account for the enthalpy of solidification?”

      That is correct and one would see similar distortions when water vapour reaches the condensation point. I considered those aspects but at this stage I only want to get the most basic concepts across.

      I could not realistically show all such detail without generating confusion.

      77

      • #

        Stephen Wilde October 24, 2015 at 6:13 pm

        bobl,

        (“I am having a hard time with the vertical displacement, (The green arrow) the parcel of air has a particular (average) kinetic energy and Potential energy, if the parcel falls then the change in PE must go somewhere either the air will heat up (KE increase) or the energy must be exchanged (it can’t just disappear). Where does it go.”)

        “I mention that the temperature of the falling air is kept stable despite the decrease in PE (and increase in KE) because it remains in contact with the falling slope of the tropopause and so can freely mix with the stratospheric air above in a diabatic energy exchange.”

        What falling slope? What is a falling slope? Your Adiabatic air parcel new is no longer adiabatic but now mixes with the stratosphere. Do you have more of what you are smoking?

        “In effect.the extra heat (from radiative gases) in the ascending column leaks up into the stratosphere whilst the cold air slides laterally and downward.”

        How and why is there any extra heat in radiative gasses? How is such heat characterized? Do you write of sensible or latent heat? Have you heard of either? Where is evidence of cold air sliding laterally and downward? Do you actually1 get away with such nonsense in front of a judge?

        “I could not realistically show all such detail without generating confusion.”

        So very kind of you!! There is no realistically or anything physically evident in any of your fantasy! 🙁

        “Let’s start by looking closely at the tropopause, which is known to undulate upwards above rising air masses and downwards above falling air masses.”

        A nice passive ‘this is known” by whom? Your disgraced CCC?

        “The height of the tropopause is set by ozone in the stratosphere reacting directly with incoming solar energy so as to create a temperature inversion that blocks further upward convection.”

        There is no evidence that ozone affects the tropopause whatsoever. More CCC fantasy! Forced convection in the tropics continues well above the whole extent of the tropopause the lowering temperatures block nothing. 🙁

        213

      • #
        bobl

        Thank you Stephen

        Will, what Stephen has done seems right to me . I think your criticism is a bit strong, but you have made several clarifying points for me.

        1. The temperature of condensation/freezing is probably modified by the low air pressure, though I’d expect the freezing point to be higher at lower pressure? Like when water vapour is frozen by the lowering of air pressure over the wing of an aircraft.

        2. I think you are right when you say that the parcel of air can no longer be adiabatic if it is mixing and exchanging energy with the stratosphere.

        3. Will, astute observation re the incomplete condensation of H2O, but to me that raises another question, if the vapour is condensed to a colloid, the latent heat is released and emitted, incoming sunlight may then break the weak polar forces holding these aggregates together re-evaporate the droplets, converting incoming sunlight to latent heat again to be emitted sometime later. Surely that’s a cooling influence even for stratospheric clouds that are supposed to warm?

        On the whole though these are minor qualifications to Stephens article.

        Stephen, thanks for the note about the omission of the latent heat exchange points, it did not make sense to me that these wouldn’t show up.

        90

        • #

          bobl October 24, 2015 at 8:11 pm

          “Will, what Stephen has done seems right to me . I think your criticism is a bit strong, but you have made several clarifying points for me.”

          I am not criticizing Stephen except that he spouts 1890 meteorology. It is meteorology itself that brought on CAGW.
          Meteorologists have been fooling both themselves and others for so long that that brand if nonsense is still taught in universities by arrogant academics. At the same time they remain are unaware that they are being fooled an manipulated by masters of scam, using the same meteorological fantasy. There is no science in meteorology. If earthlings are ever to actually learn about this atmosphere; all of the meteorological fantasy must be flushed!!! Stephen tries to delay that flushing. 🙁

          “1. The temperature of condensation/freezing is probably modified by the low air pressure, though I’d expect the freezing point to be higher at lower pressure? Like when water vapour is frozen by the lowering of air pressure over the wing of an aircraft.”
          Rime ice forms on all surfaces of an aircraft even on the underside of the wings.

          “2. I think you are right when you say that the parcel of air can no longer be adiabatic if it is mixing and exchanging energy with the stratosphere.”

          There is no place in the atmosphere that can be adiabatic! Meteorology especially the ancient form uses such term only because they sound learned and scientific. All part of the scamming process.

          “3. Will, astute observation re the incomplete condensation of H2O, but to me that raises another question, if the vapour is condensed to a colloid, the latent heat is released and emitted, incoming sunlight may then break the weak polar forces holding these aggregates together re-evaporate the droplets, converting incoming sunlight to latent heat again to be emitted sometime later. Surely that’s a cooling influence even for stratospheric clouds that are supposed to warm?:”

          That goes on in the atmosphere continually. You must define what ‘you’ mean by cooling! to Stephen and the rest of the CCC it is but a lowering in temperature, except when it is a transfer of heat from.The CCC will never say as that is part of the scam!
          I find the back and forth between Stephen and Kristain like two kids in a sand box! Both are completely wrong for different reasons. Stephen can read but interprets the pre 1900 meteorology as written then i.e. no EMR but the stratosphere is cold so that is where the heat goes FULL STOP. Kristain believes his equations δU = Q-W to be ‘the physical’ rather tand sometimes a compact expression for part of the physical under consideration. Neither can clearly express the differences in power, energy, heat, force, work; in words all are but symbols always correct but with no meaning! 🙁

          111

  • #
    Peter C

    The lapse rates graph a for Radiative (Green House) gas looks like a refrigeration cycle to me.

    20

  • #
    RB

    Kind of makes it stupid to blame extremely hot days on AGW even if it were significant. The frequency shouldn’t have changed much since the 50s and they shouldn’t even be the full half a degree warmer than usual because the global average went up by half a degree.

    20

  • #
    bobl

    PS to give a sense of scale 10,000 sq metres is roughly the size of a bigger high school gymnasium (one that can hold a basketball court) with a 10m ceiling height. Open the doors and windows and the roof vents, put your single LED Christmas light in the middle then go to the wall and see if you can measure the temperature difference.

    50

  • #

    “It is established science that convective adjustments can stabilise or neutralise radiative imbalances:”

    Nice lawyerly passive voice! I didn’t say that they did. There is no such established science! This is but meteorological fantasy. Does not rise to the level of conjecture. Even the concept of radiative imbalances is but part of that fantasy!

    “Radiative equilibrium profile could be unstable; convection restores it to stability (or neutrality)”

    Could be, might be, may be. Earth’s radiative (only) equilibrium lapse rate would be 14 to 17 Celsius/km. Meteorologists consider such to be unstable, with no reason given.

    “Note that the hydrostatic equation depicts the vertical balance of force for a piece of fluid at rest.”

    Piece of fluid??? Give me a break! Where? When? What?

    The Earth’s oceans and atmosphere are both fluids Only the easily compressible atmosphere has a pressure that remains 1.4 Newton meters times the local mass density of that atmosphere. Such requires that no part of this atmosphere exhibit any weight. Such also requires that no part of this atmosphere have any gravitational potential energy whatsoever, independent of altitude!!

    “The balance is between the upward pressure gradient force and downward gravitational force.”

    gravity does that to an atmosphere, but quite independent of local atmospheric mass or density.

    The hydrostatic equation is the vertical component of the momentum equation (Newton’s equation of motion) for the fluid parcel.” when the forces are in prefect balance and the net acceleration = 0.”

    If part of the atmosphere is stationary to the Earth’s rotating reference frame, as is gravity, that atmospheric mass expresses no momentum. If in motion that momentum is unaffected by gravity.
    Do you have a point?

    “It appears that those climate scientists who apply the radiative gases theory of climate change have overlooked the means by which convection neutralizes radiative imbalances.”

    Please note: It is the meteorologists (now climate scientists) that invented the fantasy radiation imbalances just to scam you with the fantasy CAGW! If you find incoherent fantasy scams to be entertaining Lawyer Stephen is your guy!
    Gee only paragraph three! Much more to go to skewer, lance, perforate, pierce, prick, punch: the now disgraced meteorology!! 😉

    513

    • #
      Peter C

      Earth’s radiative (only) equilibrium lapse rate would be 14 to 17 Celsius/km

      Really?

      10

      • #

        Peter C October 24, 2015 at 3:23 pm · Reply

        (“Earth’s radiative (only) equilibrium lapse rate would be 14 to 17 Celsius/km “)

        “Really?”

        Best calculation! Really really hard to stop everything else!
        This is the tropospheric temperature gradient if only surface radiation itself re-thermalized the airborne emitting molecules. Without Earth’s mobile atmospheric mass, equatorial temperatures and absolute lapse would be much higher while polar would be even less. The gravitational lapse, shifts sensible heat outward by some unknown means. Convection and latent heat assist greatly.
        All the best! -will-

        211

        • #
          bobl

          Gravitation alone without energy equalisation would result in 9.8 deg per km. This is lowered because of the massive energy transport from the surface to the tropopause. A lapse rate greater than 9.8 implies energy transport (convection) with the gravitational well, that is from top to bottom, cold to hot.

          20

          • #

            “A lapse rate greater than 9.8 implies energy transport (convection) with the gravitational well, that is from top to bottom, cold to hot.”

            That is the Claim from the CCC. No such physical lapse rate has been observed. Nor does radiative exitance determine any lapse rate. The 14-17°C/km. is a calculation without any gravitational lapse rate but only via radiative exitance. This was done only to show conclusively that surface temperature is not determined by surface radiative exitance on any body with an atmospheric pressure above 20kPa. Your CCC deliberately ignores that also. Surface EMR is never absorbed by any atmospheric gas in the sense of increasing temperature at that altitude. The temperature at every altitude is above that for radiative equilibrium.
            At or above radiative equilibrium temperature the mass is all ready transferring to colder more than can be absorbed radiatively from a higher temperature. See all of the Dr. Gus Kirchhoff excellent writings on such!

            312

  • #
    Steve McDonald

    The most powerful storm ever recorded.

    The ABC.

    The strongest ever recorded hurricane ever recorded.

    CNN.

    20

  • #
    Steve McDonald

    Paris we need a 1 dictator planet to stop this horrendous catastrophe ever happening again.

    Then I can be richer than the Clintons.

    10

  • #
    Reed Coray

    Maybe I missed it, but in addition to convection as described above, (a) the process of water evaporation from the Earth’s oceans and seas removes energy from the Earth’s surface, (b) convection moves the moist air to higher cooler altitudes, (c) where the moisture condenses back into a liquid giving up heat in the process. Finally, (d) the condensed liquid returns to the Earth’s surface via precipitation. This is exactly the principle of your car air conditioner. A cyclical process is performed where evaporation of a refrigerant occurs in a chamber thereby reducing the temperature of the chamber. The refrigerant in its gaseous state is moved to a second chamber where it is compressed to a pressure sufficient to convert the refrigerant back into liquid form thereby increasing the temperature of the second chamber. Because the temperature of the second chamber is higher than the surrounding environment, the heat in the second chamber is dissipated to the environment. It’s true that this cyclical process requires work be performed so that as a system the net effect is to add heat to the system. That’s why your room air conditioner needs to vent heated air away from the room being cooled. However, the temperature of the first chamber is lower than the ambient background environment. Thus, with sufficient thermal insolation, objects in contact with the first chamber are cooled and objects in contact with the second chamber are heated.

    In the ocean evaporation/condensation/precipitation cycle, the ocean is in contact with the first chamber and the air at high altitudes is in contact with the second chamber. Thus, in addition to heat transfer via simple convection, it seems to me that via the evaporation/condensation/precipitation cycle a tremendous amount of heat is transferred from the Earth’s surface to high altitudes where it can more efficiently be radiated to space. Bottom line, the cyclical process of evaporation, upwelling, condensation, precipitation seem to me to on balance cool the surface of the Earth. A back-of-the-envelope calculation shows that heat via ocean/sea evaporation is removed from the oceans/seas at an average rate of 3.2×10^16 watts which is about 18% of the average solar power incident on the Earth at the top of the Earth’s atmosphere. Now I don’t claim this cooling effect is greater than the heating effect of absorbing Earth surface emitted IR radiation, so I’m still open to the idea that greenhouse gases might induce a net surface temperature rise. However, I’m with Jo and Steven Wilde, in my opinion the AGW community has given too much of its attention to radiation at the expense of other means of heat transfer

    Now to the best of my knowledge atmospheric CO2 doesn’t go through a change-of-state process as it rises and falls. However, all this implies is that there are greenhouse gases and then there are greenhouse gases. In fact, one of the aspects of the whole AGW argument that has bugged me from the start is the similar treatment of all greenhouse gases. Maybe from a radiative point of view, such a treatment is warranted. But from of process of heat transfer, nothing could be further from the truth.

    100

    • #
      Bob Fernley-Jones

      @ Reed Coray,
      Yep, right,
      A few years ago I asked Roy Spencer why it was that the IPCC (Kevin Trenberth) claimed in their authoritative Earth’s Energy Budget diagram, that the largest source of energy loss from the surface is from evapo-transpiration, and yet he, et al, only seemed to talk radiation. Paraphrasing; he admitted that yes (what he termed convection) was important but everyone was focussed on radiation.

      61

      • #
        Reed Coray

        Bob, I’m a little confused regarding who he is in your above comment. Is he Dr. Spencer or Dr. Trenberth? I think he refers to Dr. Spencer. If I replace he with Dr. Spencer in your comment, would it still represent your thoughts?

        10

    • #
      KinkyKeith

      Good comment Reed.

      Excellent analogy.

      Some time ago I had an exchange with Mydogsgotno nose here about the CO2 scam and water: water is important

      http://joannenova.com.au/2012/01/dr-david-evans-the-skeptics-case/#comment-938357

      KK

      31

  • #
    thingadonta

    Is the same sort of thing as the quite unusual property of water being endothermic when evaporated?

    In other words, the evaporation of water cools the surrounding air/surface environment, which is why we sweat. This is very unusual for such processes/substances in general (most such processes are exothermic). I have always thought, since water vapour/evaporation is such a large part of the atmosphere/climate/clouds, that this might have some unusual effects, but not sure it applies to the above.

    20

  • #
    Mack

    Looking at the “Greenhouse gas” diagrams…the things I want to dispute are why are the gases which arrive back at the surface hotter than when they left?
    How is that the net effect of these gases is that they pick up heat from the atmosphere and bring it back down hotter… to warm the surface?
    You’ve got the little arrow, going back to zero, along the temperature axis, the wrong colour…blue..It should be progressively red…not blue.
    So you’ve got a little confusion (or deception) with the words…”warming at the surface” alongside this blue arrow.
    Is the gas being warmed by surface…or the surface being warmed by the gas.
    The latter is the answer if you believe in a “greenhouse effect”
    Here’s another sort of theory, which is worthy of thought when we consider the role of convection. In the case of CO2..a non-condensing heavy gas. WRT these, might not the whole lower atmosphere act as a heat pump.? ie we have the forces (work input)of convection which would hoist the hot gas to a higher altitude..it loses the heat…and then the weight of the gas to bring it back down to the surface..pick up more heat..round and round..etc etc. ie the whole atmosphere as a refrigerant.
    Makes more sense.

    31

    • #
      Peter C

      Stephen Wilde is saying that the descending gas arrives at the surface cooler than when it left. It then gets warmed by the surface before it goes up again, ie a refrigeration cycle

      70

    • #
      Mack

      Yes, I see now Peter C. The temperature axis might have confused me a bit. To avoid confusion Stephen, should change that little blue arrow at the surface to red…and also the wording…Warming at Surface…to red, also.

      10

    • #
      Mack

      Yes, I see now Peter C. The temperature axis might have confused me a bit. To avoid confusion Stephen, should change that little blue arrow at the surface to red…and also the wording…Warming at Surface…to red, also.
      Actually the whole graph doesn’t make sense.You’ve got this chunk of gas glued to the surface before convection decides to give it the heave-ho skywards…methinks.
      Global Warming occurring somewhere round our ankles.

      10

      • #

        Mack,

        It is a purple arrow to denote the combination of cool (blue) at one end and warm (red) at the other.

        That chunk of gas rises slowly as it moves across from the base of high pressure to the base of low preassure. It moves across the lines of barometric pressure which slowly transition from high to low.

        50

  • #
    Konrad

    “Can convection neutralize the effect of greenhouse gases?”

    The answer to this ambiguous question is both yes and no.

    First the use of the NewSpeak term “Greenhouse gas” pre-supposes the net effect of radiative gases in our atmosphere is atmospheric and surface warming.

    If the question is “can all processes involved in tropospheric convective circulation offset any net warming of near surface atmosphere and surface?”, then the answer is an emphatic YES.

    Our convective atmosphere acts to cool the solar heated surface of the planet during the day and has a minor role in slowing the cooling rate in the later part of the night. (an exception to this rule is the radiative night inversion layer, here due to still conditions and reduced radiative gases (clear skies) the atmosphere fails at slowing the night cooling rate.) But the NET effect of our radiative cooled atmosphere is surface cooling.

    Stephen got the right answer way back in 2011 at Talkshop. Set a pot of water above a gas burner and bring it to boil on medium heat. The water will rise to 100C and lose mass and energy through steam. Turn the gas to high. Does the water temp exceed 100C? No, all you get is more vigorous convection and more steam. Our atmosphere below the troposphere is boiling and steaming, only the “steam” from our atmosphere is long-wave infra-red radiation.

    Which brings me to the Turnbull in the middle of the sandwich. Stephen’s figure 2 is the problem, a big problem. Here Stephen commits the egregious crimes of the Climastrologists. He invokes “immaculate convention” ie: strong vertical tropospheric convective circulation without energy loss at altitude. (NO! Adiabatic cooling is NOT energy loss). For that “emphatic YES”, all processes involved in tropospheric convective circulation must include “radiative subsidence”.

    So essentially Stephen is correct. The convective cooling of the surface just speeds up in response to any attempt by atmospheric LWIR to slow its composite cooling rate. Double, triple or quadruple the amount of CO2 in the atmosphere. What happens? Time after dawn to airmass breakaway from the SBL (surface boundary layer) was advanced by “I can’t measure how few seconds”.

    Stephen may ignore radiative cooling of the atmosphere, but he is still correct. Try to warm the surface or near surface atmosphere, and convective cooling processes will just accelerate. H2O sets the temperature of our ocean planet in response to solar orbit and solar spectral variance*.

    *PS. (For David). No, Svensmark won’t do it. The hypothesis is correct, the physics works, but the power is not there. Solar spectral variance is the most likely answer. But linear calculation will never give you the answer.

    615

    • #

      Hi Konrad,

      I know we disagree as to whether or not radiative gases are needed to cause cooling with height but I am glad to see that you acknowledge that it isn’t critical for the basic scenario.

      As with the boiling water analogy, the effect of distortions of the thermal profile in the vertical column is only to enhance turbulence at the top boundary without changing surface temperature.

      I remain of the view that cooling with height is a result of converting KE to PE during uplift and radiative loss is not then necessary to establish the lapse rate and convective overturning.

      812

      • #

        Where does the latent of evaporation go upon atmospheric condensation, without sizable radiative exitance to space? This latent heat is mostly from direct insolation in the atmosphere. the surface need not be involved. This is the energy within all forms of convection including vortices that powers almost all interesting weather but is ignored by your disgraced CCC!

        614

    • #

      Konrad, you say:

      NO! Adiabatic cooling is NOT energy loss

      For the rising air mass it is.

      03

      • #
      • #

        Adiabatic cooling during ascent is not energy loss. It is transformation of energy from KE to PE within the rising parcel.

        510

        • #

          Stephen Wilde October 26, 2015 at 6:01 am

          “Adiabatic cooling during ascent is not energy loss. It is transformation of energy from KE to PE within the rising parcel.”

          More lawyerly BS! Show the atmosphere having any gravitational PE anywhere in this atmosphere. If relieved of its upward momentum via viscosity it is not attracted downward by gravitational force. The flux transfer, not adiabatic, is radiative, always in the direction of space, and you know it. Your claim of replacement of EMR exitance by convection are as intentionally false as the nonsense by the CCC! 🙁

          010

  • #

    Konrad October 24, 2015 at 5:08 pm

    “Which brings me to the Turnbull in the middle of the sandwich. Stephen’s figure 2 is the problem, a big problem. Here Stephen commits the egregious crimes of the Climastrologists. He invokes “immaculate convention” ie: strong vertical tropospheric convective circulation without energy loss at altitude. (NO! Adiabatic cooling is NOT energy loss). For that “emphatic YES”, all processes involved in tropospheric convective circulation must include “radiative subsidence”.”

    Grinn!

    Even in the 8-13 micron window at surface pressure, optical depth is less than 6km in a New Mexico canyon with the only moisture what we peed! The atmospheric column to space in that window is 50% transmissive but only 10% of the time, as clouds cover is 90% of the sky in that band. 😉

    414

    • #
      Konrad

      Clouds? Climastrologists don’t “do” clouds! Variable atmospheric IR window? This is blasphemy as defined by the Church of Radiative Climastroloy! Have a care, heretic.

      613

    • #

      Will,

      “He invokes “immaculate convention” ie: strong vertical tropospheric convective circulation without energy loss at altitude. (NO! Adiabatic cooling is NOT energy loss)”

      Why do you consider adiabatic cooling not as effective as energy loss at altitude in creating the lapse rate slope ?

      78

      • #

        Stephen Wilde October 24, 2015 at 6:31 pm

        Will, not me That was Konrad that mentioned your immaculate convection!!

        (“He invokes “immaculate convention” ie: strong vertical tropospheric convective circulation without energy loss at altitude. (NO! Adiabatic cooling is NOT energy loss)”)

        “Why do you consider adiabatic cooling not as effective as energy loss at altitude in creating the lapse rate slope ?”

        There is absolutely nothing adiabatic about this atmosphere!
        Some vertical atmospheric mass motions can be considered isentropic as no work or entropy change is involved. The decreasing temperature with altitude remains isentropic as energetic molecules leave the volume that has the temperature.
        These molecules spontaneously rejoin that volume as it reaches higher pressure and density. Some who have not considered fantasize that the temperature drop is a loss in KE and some increase in gravitational PE from increasing altitude. Such never happens in this atmosphere.
        Atmospheric mass motions can transfer both sensible and latent heat energy between locations but the heat energy itself has no mass. and is not involved with gravity. All atmospheric mass motion is isentropic as gravity dynamically adjusts the atmosphere parameters to do just that! A huge C5A takes off becoming buoyant and all local and global parameters adjust for that.
        The details of what atmospheric mass moved to where should be a subject of inquiry for you CCC. The academics are way to busy fooling folk and making profit.
        All the best! -will-

        312

      • #
        Konrad

        “Why do you consider adiabatic cooling not as effective as energy loss at altitude in creating the lapse rate slope ?”

        Because adiabatic cooling on ascent is matched by adiabatic heating on descent?

        Because adiabatic processes are buoyancy neutral in tropospheric convective circulation?

        Because Will is smarter than me, and the insufferable bastard knows this?

        512

        • #

          Jew youst made my day! 😉 Tanks!

          12

        • #

          “Because adiabatic cooling on ascent is matched by adiabatic heating on descent?”

          It is still warmer at the bottom and cooler at the top which is the essence of a lapse rate.

          “Because adiabatic processes are buoyancy neutral in tropospheric convective circulation?”

          They are not buoyancy neutral in rising columns nor buoyancy neutral in descending columns.

          711

          • #
            Greg Goodman

            How much is the buoyancy argument affected by water vapour content?

            This sounds a lot like the mechanism of Froehm storms in reverse. There the height difference is forced be the terrain and horizontal winds. Diabatic change at altitude warms the air before it descends.

            00

  • #

    The question for me is if the transport of heat to the TOA is quicker through radiation df greenhouse / radiative gasses or through the circulation of all the atmospheric gasses.

    As we know the heat transport through radiation of the GHGs is not straight but somehow bouncing to and fro (up and down) between the molecules. IR Radiation is very quick as we know. But but if it is seriously delayed during it’s travel though the troposphere, then the atmospheric circulation must be considered and calculated as well.

    The movement of atmospheric gasses in the atmosphere is surely slower than though direct radiation. The speed of heat through GHG radiation could be defined by some characteristica:
    a) IR radiation at speed of light
    b) Delay through absorption if there is any
    c) Delay while staying in the absorbing molecule
    d) the lengt of the average travel
    e) energy loss/gain through collision with other molecules

    I’m not sure if it can be considered or calculated in this way but this is a question for a long time for me.

    I try to make it even more simple. The heat in a molecule on the earth’s surface, is it faster on TOA through atmospheric circulation or through radiation of GHGs?

    Or ist the circulation just another pipe according to David Evans model?

    20

    • #

      Johannes Herbst October 24, 2015 at 7:39 pm

      “The question for me is if the transport of heat to the TOA is quicker through radiation df greenhouse / radiative gasses or through the circulation of all the atmospheric gasses.”

      What difference does it make? Power to the planet or power from then planet are both continuous/continual process. There is no musical chairs and no waiting.

      “As we know the heat transport through radiation of the GHGs is not straight but somehow bouncing to and fro (up and down) between the molecules. IR Radiation is very quick as we know. But but if it is seriously delayed during it’s travel though the troposphere, then the atmospheric circulation must be considered and calculated as well.”

      This is another diversion by the disgraced CCC The power plant does not wait for all coal to be used up before more is ordered then later delivered.

      “Or ist the circulation just another pipe according to David Evans model?”

      Alles ist ohne Unterbrechung!
      Dr. Evans model is on radiative exitance to space not on how Earth gets power to where it needs to be to radiated to space with efficacy!. That is the job of convection.

      39

      • #

        Will,

        Yes I am aware that everything is a continous process. But the “greenhouse effect” through ghgs is related to the Delay In The Heat Escape.

        Direct radiation from the earth surface is the quickest way > speed of light.

        Convection is somehow slower – Molecules heated either by contact or radiation are moved to the TOA, where the greenhouse gasses are radiating the heat to the space background.

        But – my question – how big is the (average) delay through radiation meeting GHGs until a special amount of energy gets from the Earth surface to the TOA?

        If the delay though GHG radiation is very high (means a certain heat amount starting its’s jouney to the TOA needs a lot of time) then convection plays a more impotant role in cooling the planet.

        Writing this I am even more convinced that the convection of the air masses are another pipe which has to be considered in a climate model.

        Dr Evans Model is about regulation of the earth’s climate. So the question is still: Is air convection an important transport mean for heat to the TOA or can it be neglected?

        20

        • #

          Johannes Herbst October 25, 2015 at 8:07 am · Reply

          “Will,

          “Yes I am aware that everything is a continuous process. But the “greenhouse effect” through ghgs is related to the Delay In The Heat Escape.”

          If the heat in at the bottom is the same as the heat out at the top; Where is the delay that you accept from your brainwashing?
          What is delayed? Is the EMR wavelet leaving the surface now a different color than the ones leaving to space now? Do you have a bet on the pink one? It is the same with convection. The insolation evaporates airborne water condensate at 2400Joules/gm of H2O This takes almost 16 hours of Earth rotation for the last of this latent heat to exit to space via EMR.Simultaneously at present the very same amount last latent is exiting to space that was created by direct insolation some 16 hours ago!!!

          “Direct radiation from the earth surface is the quickest way speed of light.”

          So your CCC tells you and you somehow you believe that is important only becayuse the CCC told you so!!

          “Convection is somehow slower – Molecules heated either by contact or radiation are moved to the TOA, where the greenhouse gasses are radiating the heat to the space background.”

          How large of a bet are you making on which gets to space first?

          “But – my question – how big is the (average) delay through radiation meeting GHGs until a special amount of energy gets from the Earth surface to the TOA?”

          The last of all present insolation must leave for space just before daybreak when things start absorbing again. This is about 16 hours of delay. Has been the same for the past 4000 years. the amount of H2O or CO2 does not change that even a little.

          “If the delay though GHG radiation is very high (means a certain heat amount starting its’s jouney to the TOA needs a lot of time) then convection plays a more impotant role in cooling the planet.”

          Again how large is your bet on who gets where first?

          “Writing this I am even more convinced that the convection of the air masses are another pipe which has to be considered in a climate model.”

          Dr. Evans gets as many or few pipes as he wishes His pipes have nothing to do with convection.

          “Dr Evans Model is about regulation of the earth’s climate. So the question is still: Is air convection an important transport mean for heat to the TOA or can it be neglected?”

          Dr. Evans has no model (yet) about regulation of earths climate.
          Read his articles His current expose is only about the mistakes other models make about from what altitude EMR originates on this planet.
          All the best! -will-

          011

    • #
      gai

      Dr Robert Brown and Gallopingcamel (also a physicist) and Dr. Happer said in the troposphere the IR photon is going to heat the atmosphere via collison. CO2 is not going to start re-radiating until 11 kilometers up. Dr. Happer mentioned in his lecture, experimental data shows barely any radiation at 11 KM and that radiating is in the stratosphere ~ 47 KM above the surface.

      Dr Happer’s information is illustrated by this image the Warmists use to say ozone is a greenhouse gas. The Figure is from Uherek, 2006. They even say it “show how carbon dioxide is cooling the stratosphere.” The black dotted line is the tropopause and you can see water is dumping energy just under the tropopause (the pink splotches surrounded by dark blue) while CO2 is dumping energy from just above the tropopause and up (the big yellow streak on the left) just as Dr. Happer, Dr Brown and the Camel stated. Ozone is the smaller yellow streak on the right.

      The legend with the illustration:

      Figure 2.15: Stratospheric cooling rates: The picture shows how water, carbon dioxide and ozone contribute to longwave cooling in the stratosphere. Colors from blue through red, yellow and to green show increasing cooling, grey areas show warming of the stratosphere. The tropopause is shown as dotted line (the troposphere below and the stratosphere above). For CO2 it is obvious that there is no cooling in the troposphere, but a strong cooling effect in the stratosphere. Ozone, on the other hand, cools the upper stratosphere but warms the lower stratosphere. (ibid)

      What is NOT mentions is where CO2 is active and that is NOT in the troposphere.

      00

  • #

    So he’s back. Stephen is once again allowed to promote his stubbornly flawed physical theories on how atmospheric processes work …

    1) He doesn’t even remotely understand the 1st Law of Thermodynamics (dU = dQ – dW) and how it relates directly (was in fact derived from the analysis of) “adiabatic processes”.

    2) He is incapable of distinguishing between energy transferred as HEAT [Q] and as WORK [W], thinking that any energy transfer that changes the temperature of the exchanging systems is automatically a HEAT transfer. It’s not. This is what the 1st Law expression above is telling us directly!

    3) This inability, however, leads him to fundamentally misinterpret the “adiabatic process”. He even quotes and highlights a passage explaining specifically (and correctly) what actually happens, but goes on to say with a wave of his hand that any ‘work transfer’ would be DIAbatic. Because he is still (even after having been shown to be wrong a hundred times over) firm in his belief that a transfer of energy in the form of WORK is really a HEAT transfer, if it ends up changing the temperature of the exchanging systems. Once again: It’s not. Basic thermodynamics.

    4) Stephen also continues to wallow inside that weird conceptual framework of stratospheric ozone somehow ‘blocking’ upward tropospheric convection like a ceiling, and that is even after having described how upward-moving columns of air basically push the tropopause up and downward-flowing air pulls it down, while also showing a chart of global convection and cloud tops vs. tropopause height, where the tropical tropopause is several kilometres higher than the mid-latitude ones, an obvious pattern of ‘heating-height coupling’ that is also evident in the altitude difference between the summer and winter tropopause in the temperate and polar parts of the world. Upward momentum in the troposphere is what determines the tropopause height. The lower stratosphere and its ozone are where they are because that’s where the tropospheric uplift keeps them. If the global surface were to cool, the troposphere would naturally contract, convective uplift reduce. Do you think the lower stratosphere would still remain in place? Or do you think it would follow the tropopause down? Same if the surface were to warm. Would the lower stratosphere keep the troposphere contained within the same space, blocking further upward movement? Or would it just move higher with the rising tropopause?

    197

    • #

      Kristian,

      You have been told many times by me and others that adiabatic means no heat in and no heat out, period, no exceptions.

      Saying it is transferred AS WORK does not get around that problem.

      The point about stratospheric ozone is well established, go look it up.

      Please refrain from derailing this thread.

      1013

      • #

        Stephen Wilde October 24, 2015 at 8:26 pm

        “Kristian, You have been told many times by me and others that adiabatic means no heat in and no heat out, period, no exceptions.’

        Only to the disgraced CCC! To all others adiabatic means no transfer of anything. All remains within the volume compressed or expanded. All heat, all pressure, all mass, all energy; even thoughts and desires must remain within.

        “Saying it is transferred AS WORK does not get around that problem.”

        What can that possibly mean for adiabatic. This atmosphere can have no adiabatic. Almost all is free to diffuse in any direction. Spontaneous vertical heat diffusion and and mass flow is somewhat restricted and definitely enforced by gravity!

        “The point about stratospheric ozone is well established, go look it up.”

        Only to you and other disgraced CCC!

        “Please refrain from derailing this thread.

        You Stephen continually derail your own thread because only fantasy no science!

        514

      • #

        Sigh. Same old, same old. You live in a box, Stephen. A tiny one.

        176

      • #
        Just-A-Guy

        Stephen Wilde,

        You wrote:

        Kristian,

        You have been told many times by me and others that adiabatic means no heat in and no heat out, period, no exceptions.

        Saying it is transferred AS WORK does not get around that problem.

        Yes it does. Work is not heat and heat is not work. In fact, there is no problem that requires ‘getting around’. You, Stephen, can learn about the difference between energy transfered as heat and energy transfered as work by reading Work: the transfer of mechanical energy.*

        From the article:

        The law of conservation of energy guarantees that all the chemical energy in the gasoline will reappear in some form, but not necessarily in a form that is useful for doing farm work. Tractors, like cars, are extremely inefficient, and typically 90% of the energy they consume is converted directly into heat, which is carried away by the exhaust and the air flowing over the radiator. We wish to distinguish the energy that comes out directly as heat from the energy that serves to accelerate a trailer or to plow a field, so we define a technical meaning of the ordinary word “work” to express the distinction:

        Work is the amount of energy transferred into or out of a system, not counting energy transferred by heat conduction. . .

        The conduction of heat is to be distinguished from heating by friction. When a hot potato heats up your hands by conduction, the energy transfer occurs without any force, but when friction heats your car’s brake shoes, there is a force involved. The transfer of energy with and without a force are measured by completely different methods, so we wish to include heat transfer by frictional heating under the definition of work, but not heat transfer by conduction. The definition of work could thus be restated as the amount of energy transferred by forces.

        The lack of understanding the difference between heat and work propagates throughout your entire thesis. Wikipedia also provides additional confirmation of this fundamental difference as it applies to an adiabatic process.

        From the article:

        An adiabatic process is one that occurs without transfer of heat between a thermodynamic system and its surroundings; energy is transferred only as work.[1][2] The adiabatic process provides a rigorous conceptual basis for the theory used to expound the first law of thermodynamics, and as such it is a key concept in thermodynamics.

        [1]Carathéodory, C. (1909). Untersuchungen über die Grundlagen der Thermodynamik, Mathematische Annalen, 67: 355–386, doi:10.1007/BF01450409. A translation may be found here.
        [2]Bailyn, M. (1994). A Survey of Thermodynamics, American Institute of Physics Press, New York, ISBN 0-88318-797-3, p. 21.

        While the second source is behind a paywall, you have to buy the book because there is no option to preview, the first source is avilable as a PDF which you can download and read at your leisure.

        Abe
        * Towards the end of this article the author tries to convince the reader of the increased efficiency of electric cars due to the recycling of the energy used for the brakes. He ignores the fact that while this may be true, the amount of energy expended to propel the car is not recycled and is lost forever from the system. That energy must be replaced by recharging the batteries which, of course, must be done by burning fossil fuels. AGW ™ is alive and well almost everywhere you look. 😉

        010

    • #

      Kristian October 24, 2015 at 7:58 pm

      “4) Stephen also continues to wallow inside that weird conceptual framework of stratospheric ozone somehow ‘blocking’ upward tropospheric convection like a ceiling, and that is even…. The lower stratosphere and its ozone are where they are because that’s where the tropospheric uplift keeps them. If the global surface were to cool, the troposphere would naturally contract, convective uplift reduce. Do you think the lower stratosphere would still remain in place? Or do you think it would follow the tropopause down? Same if the surface were to warm. Would the lower stratosphere keep the troposphere contained within the same space, blocking further upward movement? Or would it just move higher with the rising tropopause?”

      Good questions! This is the only part that atmospheric CO2 could affect The CO2 molecule is almost as weird as the H2O molecule.
      Much much more could, perhaps, maybe, sometimes affect lapse rate. So far the stuff is inert, but makes a wonderful dry cleaning agent above critical! Even removes caffeine from coffee beans to be later added to energy soda! For profit!
      BTW: Tomato plants are in love with the stuff!
      All the best! -will-

      212

    • #
      Just-A-Guy

      Kristian,

      You wrote:

      3) This inability, however, leads him to fundamentally misinterpret the “adiabatic process”. He even quotes and highlights a passage explaining specifically (and correctly) what actually happens, but goes on to say with a wave of his hand that any ‘work transfer’ would be DIAbatic.

      The point you’re making about the misunderstanding of an adiabatic process is correct. Unfortunately, the quote from that article only describes the end result of the convective process correctly. The description of the mechanism is way wrong.

      From that quote:

      As an air parcel rises, it moves into an environment of increasingly lower pressure (remember that pressure decreases with increasing altitude). In order to equalize the pressure difference between the the rising parcel and its new environment, air molecules inside the higher-pressure air parcel push out the sides of the parcel. That requires molecules to do work, which results in a loss of kinetic energy.

      The sentence in bold italics is the problem. Cause and effect are reversed in this explanation. Yes, the expanding parcel of air expends some kinetic energy as work. But the air parcel first expands as heat is transferd from the surface by conduction. This expansion causes lower molecular density giving it more bouyancy, making it lighter. The surrounding air is denser and therefore heavier and it’s the weight of the atmosphere which pushes the lighter air parcel up from underneath. Hot air rises not because of it’s heat content but because of it’s lower density relative to it’s surroundings.

      In fact, you don’t even need any heat at all in order to cause an object to rise in the atmosphere. A balloon filled with helium will rise because helium is lighter than the surrounding atmosphere per given volume. And it rises by virtue of the heavier atmosphere pushing it up from underneath.

      There are other obvious errors in that article but I’ll leave those for later comments. (Maybe only one or two).

      Abe

      010

      • #

        Thanks Abe!
        Can you explain how any atmospheric mass motion can be adiabatic with all molecules free to come and go as they please? I am not writing of something in the atmosphere but the atmosphere itself!
        Isentropic I can see, but no contained mass is present.
        See my #28.1.1.1 in this thread.
        All the best! -will-

        011

      • #

        Just-A-Guy,

        You appear to share Kristian’s misunderstanding.

        In an adiabatic process no heat or energy crosses the parcel boundary.

        Molecules of gas, having mass, are pulled towards a centre of gravity i.e. downwards.

        If the molecules are at absolute zero then they will settle on the surface.

        If the molecules are above absolute zero then the kinetic energy present will create an upward pressure gradient within the gas which will oppose the downward force of gravity.

        The gas will expand from surface upwards to a volume where that upward pressure gradient is balanced with the downward force of gravity and then it will stop expanding.

        The higher the temperature the more it will expand up and away from the surface.

        If one then introduces uneven surface temperatures the warmer molecules have the ability rise higher than cooler molecules and since uneven surface heating involves multiple molecules at a given location the warmer lighter molecules form themselves into parcels and stream upwards to initiate convective overturning.

        Thus a warmer lighter parcel will rise up against gravity spontaneously in the first instance. It does not need to be pushed up by colder denser gases around it.

        If one then introduces gases that are lighter at the same temperature then one doesn’t even need uneven surface temperatures to induce uplift. The lighter gases simply rise higher against gravity quite spontaneously at the same surface temperature.

        More kinetic energy or lighter weight results in gases rising higher against gravity to a higher point of hydrostatic equilibrium.

        Now, since density declines with height the rising gases cool with height due to expansion into the less dense region above. No energy is lost, there is just a conversion of KE to PE during uplift and because the higher air is colder it contracts and becomes denser and heavier so that it can be pushed aside by the warmer air coming up from below and descends again in another location, warming as it does so and there is then a constant convective overturning cycle for so long as kinetic energy can be acquired from the surface by conduction.

        There is no need for GHGs in that scenario hence the impossibility of an isothermal atmosphere developing.

        Nor is there any need for upward pushing from gases below or at the sides of a rising parcel.

        This type of buoyancy is separate from the type of buoyancy that arises when there is a liquid surface beneath for gases or solids to interact with. In that case the weight of the mass above the liquid surface can distort that surface in an entirely different process.

        Buoyancy within gases above a solid immobile surface is completely distinct from buoyancy at the interface between two deformable substances or between a solid and a deformable surface

        This is all irrelevant to my thesis anyway since it does not affect the observation that falling air warms at a specific rate so that a lowered tropopause reduces warming by compression during the descent.

        The comments of you, Kristian and Will are just attempts at misdirection of readers.

        510

        • #
          Just-A-Guy

          Stephen Wilde,

          You wrote:

          The comments of you, Kristian and Will are just attempts at misdirection of readers.

          No. We’re simply pointing out that you don’t have a clear conception of what adiabatic means.

          You also seem to think that the transfer of energy as work and the transfer of energy as heat are one and the same. They are not. This lack of understanding regarding basic thermodynamic principles is what leads to the misconception of the adiabatic process.

          Work is not heat and heat is not work.

          You were given three sources that explain the difference between these two fundamentally different phenomenon and you refuse to respond to those sources with any rational and coherent argument. You simply restate your misconception and continue to ignore the facts being presented to you.

          For your convenience, here are the referenced quotes once again:

          From the article Work: the transfer of mechanical energy:

          Work is the amount of energy transferred into or out of a system, not counting energy transferred by heat conduction.

          From the article describing the adiabatic process on wikipedia:

          An adiabatic process is one that occurs without transfer of heat between a thermodynamic system and its surroundings; energy is transferred only as work.[1][2] The adiabatic process provides a rigorous conceptual basis for the theory used to expound the first law of thermodynamics, and as such it is a key concept in thermodynamics.

          And the sources for that statement are:

          [1]Carathéodory, C. (1909). Untersuchungen über die Grundlagen der Thermodynamik, Mathematische Annalen, 67: 355–386, doi:10.1007/BF01450409. A translation may be found here.
          [2]Bailyn, M. (1994). A Survey of Thermodynamics, American Institute of Physics Press, New York, ISBN 0-88318-797-3, p. 21.

          If you disagree with what these sources are saying, then address the errors you perceive in their explanations directly.

          Abe

          26

          • #

            Just-A-Guy October 26, 2015 at 7:30 am

            “Stephen Wilde, You wrote:Just-A-Guy
            October 26, 2015 at 7:30 am · Reply

            “Stephen Wilde, You wrote: (‘The comments of you, Kristian and Will are just attempts at misdirection of readers.”)

            “No. We’re simply pointing out that you don’t have a clear conception of what adiabatic means.”

            That is true! Nor do you or Kristain! There is no such thing as an adiabatic process! That is but a fantasy invented by post modern physics. There is a well defined adiabatic cycle whereby in one half of that cycle mechanical energy does work upon a compressible fluid in compressing that fluid with no loss of energy whatsoever (no entropy). in the second half cycle the compressed fluid completely does mechanical work returning both the mechanical and gas to the original state. Such cannot be considered reversible as time has elapsed (somewhat) The entire cycle is considered still to be isentropic. Any atmospheric mass motion can never be considered adiabatic as there is never a specific gas (fluid) mass to be compressed or expanded. All atmospheric mass is free to disperse with any other atmospheric mass as long as the one gravitational constraint of tropospheric lapse is obeyed.
            All the best! -will-

            011

          • #

            “Work is the amount of energy transferred into or out of a system, not counting energy transferred by heat conduction.”

            Work can either transfer energy or transform it from one type to another. Work is a process and not a form of energy that can be transferred.
            If work moves energy from a rising parcel to the surrounding molecules then that is a transfer of heat and the process is diabatic.
            If work occurs between the rising parcel and gravity (or the pressure caused by gravity) then no energy moves out of the rising parcel. It is merely transformed from KE to PE.

            “An adiabatic process is one that occurs without transfer of heat between a thermodynamic system and its surroundings; energy is transferred only as work.”

            To transfer energy it is correct that work is required. In an adiabatic process energy is not transferred in or out by work but is transformed by work with or against gravity within the parcel from KE to PE in ascent or PE to KE in descent.

            “The key concept is that the temperature change of a parcel of air due to diabatic heating/cooling is INDEPENDENT from the temperature change caused by adiabatic heating or cooling. Temperature changes due to diabatic heating or cooling by themselves do not necessarily depend on if the parcel is rising or sinking.”

            from here:

            http://www.theweatherprediction.com/habyhints/33/

            Only the transformation between KE and PE is dependent on the parcel rising or sinking. All other temperature change is diabatic. Work done between molecules inside the parcel and ouside the parcel do not involve that transformation and so are diabatic.

            But none of this is critical to my thesis as long as descending air warms at the lapse rate and the tropopause height is lowered above descending columns as it is.

            510

          • #

            Hehe, as you can see, Just-A-Guy, there is absolutely no talking to Stephen Wilde. He’s in complete denial. He’s like the proverbial wall. He will interpret any quote you present to him as confirmation of his own warped ideas. Or just ignore it. So good luck convincing him …:)

            124

            • #
              Just-A-Guy

              Kristian,

              My intent is not to convince Steven of the error of his ways. I believe he will refuse to admit to any misunderstanding on his part. If I’m wrong, then all the better.

              The intent of my comments is to show where the errors are. The casual reader is my target.

              The motivation behind this comes from my conviction that the process that Steven has presented in the OP does in fact describe what’s happening in the atmosphere.

              Conduction from the surface to the atmosphere quickly leads to convection. Convective currents develop throughout the atmosphere and redistribute the energy acquired by conduction. Atmospheric pressure regulates the temperature (the temperature we actually measure with thermometers) in the Troposphere by creating a temperature gradient.

              The model and composition of the Standard Atmosphere and the various Adiabatic Lapse Rates, (dry, saturated, etc.) not only tell us all we need to know about the temperature distribution within the atmosphere, (in particular the Troposphere), but also pre-empt any real or claimed/imagined atmospheric temperature changes due to radiative transfer originating from the Earth.

              No GHE ™.

              The real problem with this whole article by Steven Wilde is not that the core idea is wrong, it’s not. The real problem is that Stevens description of the mechanisms involved is based on a very flawed comprehension on his part. This lack of comprehension shows up in his descriptions and this gives the impression that the core idea is wrong.

              That’s where the real damage is being done here. The core idea is correct. The presentation is dismal. I mean, for cryin’ out loud, he’s got Gravity Waves in his diagrams! 😮 What’s up with that?

              Abe

              39

              • #

                Just-A-Guy

                Thank you for that clarification.

                The more complex diagram showing gravity waves amongst other extraneous aspects was supplied by Jo who thought it weould help to validate what I was saying about the tropopause being higher above low pressure than above high pressure.

                My simplified Fig 1 is the diagram that really matters.

                The article was also reviewed by David who helped convert my hand dran diagrams to a more professional looking form.

                Several readers have expressed appreciation for the conceptual clarity of my post.

                If I have indeed correctly described what goes on in the atmosphere then that is a major step forward is it not?

                510

              • #

                Just-A-Guy, you say:

                My intent is not to convince Steven of the error of his ways. I believe he will refuse to admit to any misunderstanding on his part. If I’m wrong, then all the better.

                The intent of my comments is to show where the errors are. The casual reader is my target.

                Good to hear. That’s basically all we can do. Inform the casual reader.

                But then you go on to say something that I feel needs some clarification:

                Atmospheric pressure regulates the temperature (the temperature we actually measure with thermometers) in the Troposphere by creating a temperature gradient.

                The model and composition of the Standard Atmosphere and the various Adiabatic Lapse Rates, (dry, saturated, etc.) not only tell us all we need to know about the temperature distribution within the atmosphere, (in particular the Troposphere), but also pre-empt any real or claimed/imagined atmospheric temperature changes due to radiative transfer originating from the Earth.

                No GHE ™.

                I agree of course that there is no radiatively induced “greenhouse effect”. However, the lapse rate alone is not sufficient. It gives us the temperature distribution, yes. But it does not set the starting temperature, that is, the surface temp.

                Could you please clarify your position on this …?

                13

              • #
                Just-A-Guy

                Kristian,

                You wrote:

                I agree of course that there is no radiatively induced “greenhouse effect”. However, the lapse rate alone is not sufficient. It gives us the temperature distribution, yes. But it does not set the starting temperature, that is, the surface temp.

                Could you please clarify your position on this …?

                Sure. My position is that the surface temperature is set by the Sun, primarily by direct UV going right through the atmosphere (mostly) and heating the surface. The variation in the amount of Solar radiation that reaches the surface under a clear sky due to cyclical changes in the Sun’s output and cyclical changes in the distance of the Earth from the Sun, is not enough to explain all of the variation in surface temperatures that have been observed either empirically, or by proxy, so the next logical step is to look at clouds.

                Spencer, Svensmark, Shaviv, and now Evans, have all contributed to a better understanding of the role that clouds must play in regulating how much of that UV reaches the surface.

                I include Dr. Evans even though he hasn’t presented the updated version of his ‘Notch’ theory because I followed the original series of posts and found that there’s something there worth serious consideration. I don’t think I’ll be disappointed with his newer work.

                I wish they’d all drop the GHE ™ propaganda already, but that’s another story.

                Abe

                03

              • #

                Just-A-Guy, you say:

                My position is that the surface temperature is set by the Sun, primarily by direct UV going right through the atmosphere (mostly) and heating the surface. The variation in the amount of Solar radiation that reaches the surface under a clear sky due to cyclical changes in the Sun’s output and cyclical changes in the distance of the Earth from the Sun, is not enough to explain all of the variation in surface temperatures that have been observed either empirically, or by proxy, so the next logical step is to look at clouds.

                Spencer, Svensmark, Shaviv, and now Evans, have all contributed to a better understanding of the role that clouds must play in regulating how much of that UV reaches the surface.

                Cheers 🙂

                I agree to the extent that it’s obvious the Sun is our planet’s only energy/heat source of any significance, and so the actual heating of our global surface is done pretty much by the Sun alone.

                However, this does not mean that the Sun alone sets the mean surface temperature of our planet. There is heat IN and there is heat OUT. And in any thermodynamic system, steady state temperature is achieved only at the balance point between the two. How do you get there for a solar-heated global planetary surface? Without an atmosphere on top? Versus with an atmosphere on top?

                10

              • #

                Kristian
                October 28, 2015 at 1:13 am

                “However, this does not mean that the Sun alone sets the mean surface temperature of our planet. There is heat IN and there is heat OUT. And in any thermodynamic system, steady state temperature is achieved only at the balance point between the two. How do you get there for a solar-heated global planetary surface? Without an atmosphere on top? Versus with an atmosphere on top?”

                Generally the 5km altitude temperature is much more stable than the surface temperature. The locally variable lapse can then be used to determine both surface and higher altitude temperatures. I.E. Phoenix AZ generally has both a higher value temperature lapse and higher surface temperature than Mobile Al with much higher humidity.
                All the best! -will-

                07

  • #
    Steve McDonald

    We can stop the loss of our nation to people who think that they must rule us.
    Defund the A.B.C. of poor peoples money.
    Get them off the dole.

    41

  • #
    Geoff Sherrington

    Because of a deprived education, I see some complexities in these explanations that cause me to reject hypotheses too quickly. Here is a question that first arose from analogies to quantum physics, which still argues with particle versus wave theories.
    Question: Are these physical effects in P T KE etc confined to a group of particles, or do some of them move independent of parcels? Put another way, those winds of aicrat altitude that scoot along at several hundred km per hour are not seen on the ground, so there must be a change boundary between the as one goes vertically. Does that change involve the movement of a parcel of like molecules, or is there a faster portion that has a queue jump, as by radiative transfer?

    20

    • #

      Clearly there is a large horizontal movement component in convection. Some radiative transfer can then occur from the same altitude but different location. Convection can be seen from this story in 1898 to have been the cause of the heat (at one location) rather than providing rapid cooling. You describe looking up through the boundary between the conical cyclone and anti-cyclone. Like two cogs moving against each other the bulk of the air moves in ONE direction but the ground it would appear to be two different directions.
      But now I lind there is another such depression on the extreme west of Central Australia, whose progress east is blocked by a high pressure or anti-cyclonic area rising between us and New Zealand like a great mountain. In 1894 a similar obstruction rose, which delayed the weather for a week. The West and South Australian people are expecting a thunderstorm; but we are not likely to get it. On the contrary, the north winds will bring us only heat from the interior. “
      http://trove.nla.gov.au/ndp/del/article/191491117

      10

  • #

    Thanks to all who attempted to explain. I confess to being just as puzzled as before, however slowly but surely my mind is starting to grasp some of the factors we are dealing with. (I was never a whiz in math classes.)

    I have been closely watching the North Pole for years, mostly because I like the beautiful pictures of summer sea-ice at the Pole when it is hot and humid where I live, but also because I wanted to see for myself what the ice was doing, because the media was doing a pathetic job reporting. Besides seeing that the so-called “Death Spiral” was bunkum, I’ve observed some odd drops in temperature I never could explain, right in the middle of the summer, when the sun never sets.

    I figured that these drops in temperature must have something to do with heat being lost to space, as they seemed associated with summer low pressures that were weakening and filling. I ventured the idea that what went up didn’t come down. When I did this on some site (WUWT?) I proceeded to be intellectually drubbed by a NOAA meteorologist who knew roughly a thousand times as much science as I did. He basically stated air cooled to very cold temperatures at the tops of clouds, and at those low temperatures there was so little heat left in the air that little heat could be lost, (and little heat-loss was seen by satellites). Then the air simply rewarmed as it descended. It was like a yo-yo, with adiabatic cooling on the way up and adiabatic cooling on the way down.

    Ever since being drubbed to such a embarrassing degree, I’ve been timid about mentioning what my lying eyes observe, which is that rather than the air being warmed by rising and gaining latent heat by raining out moisture and then descending warmer(like a Chinook coming down from a mountain), the descending air is sometimes cooler.

    20

  • #
    Richard

    Here’s an interesting video from Salby arguing that convective feedback cools the surface temperature by 60C. Fast forward to 59:30.

    30

  • #

    I will refer to Stephen’s quote from his own source:
    https://courseware.e-education.psu.edu/public/meteo/meteo101demo/Examples/Section6p04.html

    It says:

    As an air parcel rises, it moves into an environment of increasingly lower pressure (remember that pressure decreases with increasing altitude). In order to equalize the pressure difference between the the rising parcel and its new environment, air molecules inside the higher-pressure air parcel push out the sides of the parcel. That requires molecules to do work, which results in a loss of kinetic energy. With kinetic energy expended by air molecules to push out the sides of the expanding parcel, the temperature of the air inside the parcel decreases (recall that temperature corresponds to the average kinetic energy of molecules). The energy spent by molecules to push out the sides of the parcel amounts to a flat rate of 5.5 degrees Fahrenheit per 1000 feet of ascent (10 degrees Celsius per 1 kilometer).

    (My emphasis.)

    There’s a clever animation right next to this paragraph visually representing what’s going on. You will see there how the molecules inside the parcel do work [W] on the surrounding air by pushing it aside. They are able to do this not because the parcel’s internal pressure has increased, but because the external pressure of the surrounding air has decreased. And so the work done (the amount of molecular (kinetic) energy spent expanding the parcel against the external pressure) is simply ‘net force/pressure’ times the expanded ‘distance’.

    If you have even a basic understanding of thermodynamics, energy transfers and resulting system temperature changes, you will know as a trivial fact that when one system does work on another (or its surroundings), internal energy [U] is transferred FROM the system performing the work TO the system on which the work is being performed. If not, there could be no work done, no result, no change in either.

    And this is of course directly equivalent to a transfer of heat [Q], whereby one system transfers internal energy to the other by virtue of a temperature difference between the two. The one system could not heat the other, there could be no result, no change, if energy weren’t actually transferred.

    The quote above, however, never actually states this fundamental circumstance in plain words. There could thus still arise some confusion as to where the expended molecular kinetic energy actually ends up. If, that is, you don’t have the proper background in thermodynamics. Then you know. Stephen obviously doesn’t. And thus completely misses the whole point of the “adiabatic” mechanism for temperature change.

    But he would only need to read a couple of paragraphs further down in order to reach enlightenment on this issue. It says:

    As a parcel sinks, it moves into an environment with increasingly higher pressure (…). Now air molecules surrounding the parcel perform work as they push the sides of the parcel inward (while trying to equalize pressure between the parcel and the parcel’s environment). In turn, air molecules inside the parcel are the beneficiary of the work done by the environment, gaining kinetic energy as the sides compress inward. This gain in kinetic energy results in a temperature increase inside the air parcel.

    (My emphasis.)

    This can hardly be misunderstood. Yet, Stephen insists on doing just that. He’s been told and shown a hundred times by several different people. He refuses to listen, refuses to read. He still thinks that the molecules do work against gravity only and so simply convert their kinetic energy into potential energy, no energy transferred anywhere. This is so far off the scale of physics illiteracy that it quite frankly merits no further comment.

    The “adiabatic process” functions straightforwardly according to the 1st Law of Thermodynamics, expressed here for a closed system: dU = dQ – dW.

    U is the system’s “internal energy”. The change in U is always proportional to the change in system temperature [T].
    The U, then, can change via two routes, two different kinds of energy transfer to/from the system: 1) by a transfer of energy in the form of HEAT [Q], or 2) by a transfer of energy in the form of WORK [W].

    In thermodynamics, work [W] is a collective term for all kinds of energy transfers that don’t come about simply through direct ‘heating’ (a temperature difference).

    So basically you have ‘energy transfer through heating’ [Q] and ‘all other energy transfers’ [W].

    What, then, defines an “adiabatic” process? The ‘no heat transfer’ clause. An “adiabatic process” is defined as one where no direct ‘heating’ is taking place. There is no transfer of energy as HEAT, due only to the temperature difference between systems.

    Which mathematically can simply be expressed like this: Q = 0.

    How does this condition affect the 1st Law expression above? Like this: dU = -dW.

    So the change in the system’s U is now equal to the W done by or on the system. The minus sign ensures that the more work performed BY the system (the more positive it is), the more the system U will decrease, and so the more the system temperature will drop. And the other way around …

    The work done in a traditional adiabatic process, the expansion and compression/contraction of a gas volume, is termed “pressure-volume work” (PV work), because the work is perfomed in equalising internal and external pressures by changing the volume of the gas. And so the simplest expression for the adiabatic process becomes:

    dU = -PdV

    124

    • #

      Kristian,

      The source does not say that other molecules have to be pushed aside when the parcel expands its sides outward.

      That would only be necessary if the other molecules were in a region of the same or higher pressure.

      When they are in a region of lower pressure that reduction in pressure has already moved them further apart so that no further pushing is necessary.

      It is like a retreating army on a battlefield being replaced by an advancing army.

      The advancing army does not need to push the retreating army anywhere.

      The advancing army needs to do work in the process of advancing but the work is not done against those retreating.

      On a battlefield the advancing army does work on the battlefield surface in order to advance. In the atmosphere the expanding molecules do work against the gravity field in order to expand.

      No energy in any form passes between the parcel and other molecules, otherwise the process would be diabatic and not adiabatic.

      712

    • #
      Richard

      He still thinks that the molecules do work against gravity only and so simply convert their kinetic energy into potential energy, no energy transferred anywhere.

      I don’t know anywhere near as much about this subject than a lot of people here and I may just be talking out the back of my head, but by my (admittedly novice) understanding the ‘energy’ in this circumstance is the force of gravity and hence there is no violation of the 1st law of thermodynamics. Here is my understanding: the air molecules that are falling in earth’s gravitational field and the mass of the atmosphere compresses the air molecules at the surface and it’s this compression and decrease in volume that increases temperature, as would be predicted by the Ideal Gas Law. I think a potential aid in understanding this gravitational heat mechanism are to look at the other planets such as Venus, Jupiter, Saturn, and even the sun. Jupiter, interestingly, has an estimated core temperature of around 24,000K. How is that possible when it receives so little radiation from the sun? One explanation for these high temperatures (and correct me if I am wrong but I believe it is the mainstream explanation) is that the planet has an enormous atmospheric mass – some 400 times greater than earth – and it’s estimated that if the planet’s atmospheric mass (which is mainly hydrogen) increased by 70-80 times then it would be able to generate enough gravitational pressure to reach temperatures necessary for nuclear fusion. This is why Jupiter is sometimes referred to as a ‘failed star’. All that Jupiter would need to achieve these temperatures is more mass, and gravity and compression would do the rest. In order to countervail the greater gravitational compression from the added mass the outward pressure of the gases must increase by raising the temperature (i.e. ‘hydrostatic equilibrium’).

      20

      • #
        Richard

        This is more my understanding of how the surface temperature of a planet can increase above ASR rather than addressing your quoted comment above, which re-reading again I think I misunderstood.

        20

      • #

        Richard, you say:

        (…) by my (admittedly novice) understanding the ‘energy’ in this circumstance is the force of gravity and hence there is no violation of the 1st law of thermodynamics.

        This statement makes no sense. Are you saying that the gravitational force itself is energy? How does that work?

        And who says the 1st Law is violated? Stephen simply doesn’t understand it. Different thing.

        Here is my understanding: the air molecules that are falling in earth’s gravitational field (…)

        There are no ‘air molecules falling in Earth’s gravity field’, Richard. That’s the whole point. Gravitational potential energy is not applied to individual air molecules, really does not concern them at all. It applies to the air, the atmosphere as a whole. When the centre of mass of an air mass (or of the atmosphere as a whole) for some reason moves up or down, its gravitational potential energy changes. This does NOT, however, come at the expense (or benefit) of the internal (microscopic/molecular) kinetic energy of the air/atmosphere. A solid object does not cool just from you lifting it. Even though it moves against the pull of gravity, so increases its gravitational potential energy. Likewise it doesn’t warm back as you lower it back down. So how come air cools as it lifts. It cools because it expands. Not against gravity, but against the surrounding air, against the total force (pressure) applied to it by the surrounding air molecules. Expanding against this requires thermodynamic WORK to be performed. By the air molecules inside the rising air parcel. On the air molecules outside the rising air parcel.

        It’s that simple. Stephen’s silly theory is wrong. Period. He will never admit it, of course. But it makes no difference. He is trivially wrong, and any person trained in thermodynamics sees this immediately. They have also tried to point it out to him over and over again, but have now simply given up on him. I’m most likely the last one caring …

        (…) and the mass of the atmosphere compresses the air molecules at the surface and it’s this compression and decrease in volume that increases temperature, as would be predicted by the Ideal Gas Law.

        No, it only increases temperature at the initial compression. That is what the IGL ‘predicts’. Such a particular temperature rise is not a permanent thing.

        I think a potential aid in understanding this gravitational heat mechanism are to look at the other planets such as Venus, Jupiter, Saturn, and even the sun. Jupiter, interestingly, has an estimated core temperature of around 24,000K. How is that possible when it receives so little radiation from the sun? One explanation for these high temperatures (and correct me if I am wrong but I believe it is the mainstream explanation) is that the planet has an enormous atmospheric mass – some 400 times greater than earth – and it’s estimated that if the planet’s atmospheric mass (which is mainly hydrogen) increased by 70-80 times then it would be able to generate enough gravitational pressure to reach temperatures necessary for nuclear fusion. This is why Jupiter is sometimes referred to as a ‘failed star’. All that Jupiter would need to achieve these temperatures is more mass, and gravity and compression would do the rest. In order to countervail the greater gravitational compression from the added mass the outward pressure of the gases must increase by raising the temperature (i.e. ‘hydrostatic equilibrium’).

        Richard, in a way you’re on to something here, but it is not the atmospheric compression that keeps those inner/lower regions so hot …

        47

        • #

          Kristian said:

          “It cools because it expands. Not against gravity, but against the surrounding air, against the total force (pressure) applied to it by the surrounding air molecules. Expanding against this requires thermodynamic WORK to be performed. By the air molecules inside the rising air parcel. On the air molecules outside the rising air parcel”

          The total pressure applied to the parcel by the surrounding air comes from two distinct sources:

          i) Collisional activity between molecules

          ii) The weight of higher molecules derived from the gravity field.

          Work done in relation to the former does involve a transfer of energy in and out of the parcel and that is diabatic. Collisional activity is involved.

          Work done in relation to the latter does not involve a transfer of energy in or out and that is adiabatic. Collisional activity is not involved when expanding into a region of lower pressure or contracting into a region of higher pressure. All that is needed is uplift or descent resulting from density and thus buoyancy differentials.

          Anyway, is this at all relevant to the basic proposition set out in this thread to the effect that the proposed convective adjustments can neutralise the thermal effect of radiative gases ?

          612

          • #

            Stephen Wilde October 25, 2015 at 1:55 am

            (Kristian said:

            “It cools because it expands. Not against gravity, but against the surrounding air, against the total force (pressure) applied to it by the surrounding air molecules. Expanding against this requires thermodynamic WORK to be performed. By the air molecules inside the rising air parcel. On the air molecules outside the rising air parcel”)

            There is no work done as there is no force required for the still energetic molecules to spontaneously leave your imaginary volume.
            The volume is not expanding the molecules are leaving. An 8 m^3 baggy with only one m^3 100 kPa dry atmosphere within would be full at an pressure altitude of 12.5kPa. If well insulated it would also be at a temperature approx 100°C lower than at the surface with pressure 100kPa.
            This is what i think that Steven and the rest of the CCC crowd consider to be adiabatic. They will never tell and it is hard to guess about such foolishness.

            SW “The total pressure applied to the parcel by the surrounding air comes from two distinct sources:
            i) Collisional activity between molecules
            ii) The weight of higher molecules derived from the gravity field.”

            No Part of this atmosphere expresses any weight. There also is no surround pressure deferential! All becomes isobaric at the speed of sound. In comparison your convection is not moving!

            “Work done in relation to the former does involve a transfer of energy in and out of the parcel and that is diabatic. Collisional activity is involved.”

            Stephen finally admits that the is no atmospheric adiabatic!
            He is still wrong as there is no work done! all is isentropic. Molecules leave the volume and supply (collisional transfer) power to all H2O molecules spontaneously radiating this entropy to space. 😉

            “Anyway, is this at all relevant to the basic proposition set out in this thread to the effect that the proposed convective adjustments can neutralise the thermal effect of radiative gases ?”

            Such does not ‘neutralize’ EMR exitance, presented in an informative way, they absolutely depend on it
            All the best! -will-

            09

            • #
              Just-A-Guy

              Will Janoschka,

              You wrote:

              He is still wrong as there is no work done! all is isentropic. Molecules leave the volume and supply (collisional transfer) power to all H2O molecules spontaneously radiating this entropy to space. 😉

              Yes, of course! These molecules being more energetic (having a higher translational speed), have GPS installed on the dashboard. That’s how they know to avoid collisions with any slower, less energetic molecules on the road and escape the volume they currently occupy. 😮

              They also use their advanced GPS System to avoid collisions with all other atmospheric molecules except for H2O and then plot a direct collision course so that they can deliver their cargo directly to them and only them! 😮

              Abe

              05

            • #

              Just-A-Guy October 26, 2015 at 7:25 pm ·

              “Yes, of course! These molecules being more energetic (having a higher translational speed), have GPS installed on the dashboard. That’s how they know to avoid collisions with any slower, less energetic molecules on the road and escape the volume they currently occupy. 😮 ”

              Do you really think so? Where can I get some of those GPS units for my molecules? When a molecule collides with a different mass less energetic molecule, what are the new directions and velocities of the two molecules? what is the total change in momentum that must increase temperature?
              You seem to know so much!

              “They also use their advanced GPS System to avoid collisions with all other atmospheric molecules except for H2O and then plot a direct collision course so that they can deliver their cargo directly to them and only them! 😮 Abe”

              Really? What is that mechanism? You seem to know so much!

              08

        • #

          Kristian said:

          “No, it only increases temperature at the initial compression. That is what the IGL ‘predicts’. Such a particular temperature rise is not a permanent thing.”

          If one introduces convective overturning there is an ongoing process of decompression and compression which requires a steady source of heat at the surface over and above that which is radiated to space.

          711

          • #

            Stephen Wilde October 25, 2015 at 1:58 am · Reply

            (Kristian said:
            “No, it only increases temperature at the initial compression. That is what the IGL ‘predicts’. Such a particular temperature rise is not a permanent thing.”)

            “If one introduces convective overturning there is an ongoing process of decompression and compression which requires a steady source of heat at the surface over and above that which is radiated to space.”

            Yes, Continuous insolation converted by atmospheric mass to both sensible and latent heat, reduces density, some just by molecules leaving that volume creates local positive buoyancy. There is never any compression. Because of this positive buoyancy upward atmospheric motion begins. Such motion, however, does not limit the rate of upward heat flux which is many times that of the sensible heat transfer of physical mass motion.
            Please, please, flush all meteorology, all CCC, and all academic elite that support such nonsense. Hire some that demonstrate that they can at least locate their own gluteal muscles with one or more upper appendages! 😉

            011

  • #
    Ruairi

    Alarmists may think that they know,
    All Earth’s spheres high above and below,
    But some honest reflection,
    On tropospheric convection,
    Might reveal what their models don’t show.

    30

  • #

    The article does introduce a pushing from surrounding molecules in descent but I consider that to be wrong. In reality the increasing pressure caused by gravity compresses the descending parcel without work needing to be done by the surrounding molecules. The work is done by the pressure from the gravity field working on the falling molecules.

    I think your confusion arises from a failure to distinguish between pressure caused by molecular motion and pressure caused by the weight of the molecules attributable to the gravity field.

    A diabatic process refers to an interaction between the pressure caused by molecular motions in and outside the parcel. That does involve pushing or pulling and does move heat in and out in a diabatic process.

    If the interaction is between the parcel and the pressure caused by the gravitational field then no energy or heat transfers in or out and that is an adiabatic process.

    You clearly told me long ago that you believed tha a rising parcel lost heat to the surroundings and a falling parcel gained heat from the surroundings but that is a diabatic not an adiabatic process.

    611

    • #

      “I think your confusion arises from a failure to distinguish between pressure caused by molecular motion and pressure caused by the weight of the molecules attributable to the gravity field.”
      Not sure who you are talking to but I am confused so it may as well be me. Anyhow it seems that the two can be seperated by looking at what happens over cyclic change periods.

      Given the relation between drag and air
      density, and precise measurements of satellite orbits, one may obtain extensive data
      for atmospheric density. The method is described in detail by Jacchia (1963). Extensive
      measurements (Harris and Priester, 1965) show an extremely strong daily
      variation in density at all levels above 200 km. Density varies by a factor of five during
      a day at 360 km, maximum densities occur near 1400 hours local time.

      RICHARD S. LINDZEN and SYDNEY CHAPMAN 1969
      http://www-eaps.mit.edu/faculty/lindzen/29_Atmos_Tides.pdf

      In that he has a very large section devoted to the Lunar atmospheric cycles. If you think looking at large daily change above 100Km is mad, they quote the “Society of Lunatics” not me.

      10

      • #

        Siliggy

        I was addressing Kristian who thinks that the temperature change within an adiabatically rising or falling parcel is due to work done between molecules inside the parcel and molecules outside the parcel (pressure caused by molecular motion) whereas in fact it is work done between the parcel and the gravity field (pressure caused by weight induced by gravity).

        58

        • #

          Stephen Wilde October 25, 2015 at 5:07 pm

          “I was addressing Kristian who thinks that the temperature change within an adiabatically rising or falling parcel is due to work done between molecules inside the parcel and molecules outside the parcel (pressure caused by molecular motion) whereas in fact it is work done between the parcel and the gravity field (pressure caused by weight induced by gravity).”

          Both you and Kristian seem myopic but very much into pushing an individual religion with no science.
          Stephen Can only profess from a 1900 book!
          Kristain_ Can only profess equations δU= Q-W with no understanding of the symbols!! 🙁

          This atmosphere expresses no weight whatsoever! The individual atmospheric molecules spontaneously arrange themselves such as to produce a logarithmic pressure gradient decreasing with altitude. This is accompanied by a logarithmic density gradient decreasing with altitude, having a constant isentropic exponent of 1.4 Newton-meters.
          This exponent looks like work, but is a component of gas kinetic energy! This exponent expresses as a linear temperature gradient decreasing with altitude. Such is demanded by the kinetic theory of gas. This has nothing to do with any atmospheric motion or convection. It is but the thermostatic state of a compressible fluid surrounding a more incompressible mass exhibiting gravity.
          Can either of you explain or demonstrate any ‘necessary’ work to re-locate any portion of the atmosphere in any direction within that atmosphere?
          Can either of you explain or demonstrate any possibility that any portion of the atmosphere can compress or decompress in an adiabatic manner? The first property of an adiabat is containment of a specific mass or molar quantity of gas! There can be no atmospheric parcel!
          The closest physical approach would be a defined volume, full of holes equipped with barometer and thermometer. With such one can demonstrate that upon heating the volume many energetic molecules just leave that volume to a location less noisy! Up she goes shedding both more molecules and temperature but not necessarily any rms velocity of the molecules remaining in that volume. As the remaining molecules shed sensible heat to space via EMR the pressure/temperature drop; inviting local more energetic molecules to accumulate increasing density.
          Shortly motion and reverses moving downward the volume is accumulating both molecules and temperature, even all the way back to the surface. No work was ever done. All that happened is that the sensible or latent heat with no mass got dispatched to space with maximum efficacy.
          All the best! -will-

          011

        • #

          Stephen I can see the problem and the cause. You lot will argue forever with no resolution until you see it too. Two bodies in space are supposed to be attracted to each other by gravity. This magic thing called gravity is supposed to provide the free energy to accelerate the objects together. That is the problem in a nutshell. The theory of gravity will always lead you into a free energy condundrum.
          The need for dark energy and dark matter are the same problem in a different coat. If you instead theorise gravity as an external pushing force that DOES use energy from space then all these problems go away.
          Will
          “Can either of you explain or demonstrate any ‘necessary’ work to re-locate any portion of the atmosphere in any direction within that atmosphere?”
          Can you explain how relocating anything at all can EVER be done without using energy? Your explanation is just like a lava lamp. The lava lamp will stop if you turn the energy supply off.

          02

          • #

            The necessary energy source is insolation at the surface followed by conduction and convection.
            Convection recycles that same energy up and down as long as insolation continues.

            Gravity provides no energy itself.

            Convection will stop if you turn off insolation.

            60

          • #

            Siliggy October 26, 2015 at 4:07 am

            (Will “Can either of you explain or demonstrate any ‘necessary’ work to re-locate any portion of the atmosphere in any direction within that atmosphere?”)

            “Can you explain how relocating anything at all can EVER be done without using energy? Your explanation is just like a lava lamp. The lava lamp will stop if you turn the energy supply off.”

            A marble rolling on a plane normal to gravitational attraction will continue to roll until some other force acting upon it (friction, viscosity) decelerates the mass. That surface is called isopotential. An air puck table is another example of no work required for relocation of the puck! Mass accelerations and decelerations still require work but this is different from relocation where no work (force x distance) is required. This atmosphere is an isopotential volume for any self buoyant mass (a portion of that atmosphere at any location). The pressure, density, and temperature gradients set by gravity, not only set such, but demand that conditions re-equilibrate at the speed of sound. These thermostatic conditions also force massless heat transfer at extraordinary velocities. The lapse is always maintained. The atmosphere is weightless. The lapse is part of gravity itself!
            All the best! -will-

            08

            • #

              “Mass accelerations and decelerations still require work ”
              Do you mean accelerations like 9.8M/S/S as per the following example?
              If you ripped your foot off an threw it up then work is done. Gravity will then provide the energy to overcome drag again and ACELLERATE it down into your mouth. Free storage and free energy just like wind turbines stealing convection energy from natures feebly underpowered cooling fan.

              00

    • #

      “The difference lies in the absence of water vapor in the descending column which then warms at a different rate to the cooling in ascent.”
      This makes very good sense.
      What happens over a day or longer cycle could enhance it even more while CO2 radiation going sideways (Horizontal) would reduce convection by bypassing it and making it redundant.

      30

  • #
    Roy Hogue

    I don’t have the background to enter this debate. But it is quite obvious that the climate is stable over a long period of time in spite of some severe swings of temperature (weather), cloud cover and more to the point, green house gasses. So intuitively it appears that there is some mechanism whereby the atmosphere smoothes these variations out over time. And it certainly must be the atmosphere because it’s the only thing available to do that job.

    Stephan’s hypothesis at least seems reasonable at face value, just on the basis I stated.

    Actually I’ve understood much of what he argues, even without going through all the math. This blog has been the best place in the world for a guy like me to learn. So thank you Stephen and Jo. 🙂

    62

    • #
      PeterPetrum

      Me to, Roy. The education I have received from the authors and erudite bloggers on this site is immeasurable. Thank you to you all.

      40

    • #
      Roy Hogue

      And for that I get a red thumb??? What very strange and wonderful times we live in these days. But I’d like to go back to when the kid who didn’t like me had to face me and voice his complaint in person. I could deal with that, even give it a certain amount of respect. But anonymity doesn’t impress me.

      I Suppose this will get another one. So be it.

      40

      • #
        David Cosserat

        Roy,

        I fully endorse your point. Anonymity doesn’t impress me either. And anonymous bloggers who clearly think they are geniuses but can’t spell or bother to check their mistyped responses before posting them, particularly annoy me.

        We are all here to learn – ALL of us. People who sneer and abuse others are wasting everybody’s time. You know what it reminds me of? Road rage – where people feel it’s OK to shout and scream and gesticulate from behind the safety of a windscreen.

        All the best
        David C

        20

  • #
    Franktoo (Frank)

    Steve wrote in this post: “i) In an ascending column the CO2 at lower levels is a net absorber of radiation from the ground so that the slope of the lapse rate is reduced in an ascending column which slows down the rate of convection from the surface.

    Further on, he contrasts: “At higher levels CO2 becomes a net radiator of energy to space so that the slope of the lapse rate increases again.”

    The 2LoT demands that CO2 at any altitude be a net absorber of radiation from places where it is warmer and a net emitter to places where it is colder. Until we reach the tropopause – where convection has ceased, “warmer” means below and “colder” means above.

    The surface receives about 160 W/m2 of incoming solar radiation. Net thermal radiation takes away only part: 396 (OLR) – 333 (DLR) = 63 W/m2 (net). If convection didn’t occur, the surface would warm until OLR-DLR = ASR; which some estimate would be about 350 degK. Such a high surface temperature has a lapse rate much higher than DALR or MALR, so convection flows until a marginally stable lapse rate has been produced. That leaves a surface temperate of about 288 degK. Higher in the atmosphere, there are fewer GHGs above and therefore less DLR and more net radiation and less need for convection. By the time you reach the tropopause, net radiation provides adequate cooling and there is no need for any convection. (The tropopause is higher above the tropics than elsewhere because the tropics receive more SWR.) This is the big picture: equilibrium temperature controlled by a combination of radiation and convection.

    When you analyze one particular location and don’t discuss all of these factors (OLR, DLR, convection). it is easy get mislead. The 2LoT demands that radiation flow from hot to cold and try to eliminate the lapse rate that convection produces produces by adiabatic expansion and cooling (and compression and warming). Net radiation cools the atmosphere a few degC/day, but convection replaces this loss.

    Yes, there are some minor refinements. The surface pressure is few percent lower under rising air than descending air. There are phenomena associated with weather, our rotating frame of reference, unequal distribution of incoming radiation, heterogeneous surface and clouds that create small perturbations in this big picture. The success of weather forecasting programs that we CAN properly calculate mass and heat transfer in the atmosphere until our limited knowledge of current conditions (initialization) interferes – chaos.

    142

    • #

      Franktoo (Frank) October 25, 2015 at 3:19 am
      “…which some estimate would be about 350 degK. Such a high surface temperature has a lapse rate much higher than DALR or MALR, so convection flows until a marginally stable lapse rate has been produced.”

      Why do you claim an increased lapse rate? Yes the temperature of the whole column would be increased. likewise both pressure and density gradients may have increased. Why does the temperature lapse increase when that is only a dimensionality distinction P/rho? Why do you claim that convection can change the temperature lapse?
      All the best! -will-

      07

    • #

      Franktoo(Frank) said:

      “The 2LoT demands that CO2 at any altitude be a net absorber of radiation from places where it is warmer and a net emitter to places where it is colder. Until we reach the tropopause – where convection has ceased, “warmer” means below and “colder” means above. ”

      If the tropopause is the height at which net absorption becomes net emission to space could you explain why the so called emissions height for CO2 appears to be well above the tropopause ?

      As regards the region below the tropopause please note that the relationship between absorption and emission changes as one goes up despite there being net absorption all the way up to the height where net emissions to space develop.

      Up to the point of hydrostatic balance there is cumulative movement of the environmental lapse rate (ELR) away from the DALR but then in the remainder of the vertical column there is a cumulative movement of the ELR back towards the DALR. Since the entire column is distorted to the warm side there is inevitably an excess of convective energy in the ascent column so that the tropopause pushes up above the usual tropopause height which then leads to the vertical displacement shown in green in my diagrams.

      59

  • #
    Greg Goodman

    The tropopause is the boundary where convection runs out of oomph,

    NO, the tropopause is where the temerature gradient changes from negative to positive.

    in the troposphere, temp decreases with height. This means that warm air rises leading to turbulent mixing. In the stratosphere, solar heating means temp increases with height. This stops convection dead in its tracks. It is not a question of “oomph”

    The lack of convective circulation means there is very little vertical movement leading to a stratified atmosphere: hence the name.

    22

    • #
      Greg Goodman

      The height of the tropopause is set by ozone in the stratosphere reacting directly with incoming solar energy so as to create a temperature inversion that blocks further upward convection.

      Perhaps Jo should have read the article.

      03

  • #

    Come on fess up. Who put the sun above the polar regions?
    http://www.acom.ucar.edu/start/

    10

    • #

      Not to mention the Satellite well below 100Km.
      “Objects below approximately 160 kilometers (99 mi) will experience very rapid orbital decay and altitude loss”
      https://en.wikipedia.org/wiki/Low_Earth_orbit

      Also to put gravity back in proper perspective. “Although the Earth’s pull due to gravity in LEO is not much less than on the surface of the Earth, people and objects in orbit experience weightlessness because they are in free fall.”

      10

      • #
        Greg Goodman

        Isn’t the increased convection what was supposed to cause the upper tropo ‘hot-spot’ in the tropics? You know, that one that was barely detectable.

        Isn’t just the sort of increased convection described in this article what climatologists expected and did not find?

        41

  • #
    Svend Ferdinandsen

    It looks like http://hockeyschtick.blogspot.dk/
    He has some points, but in my opinion runs out of a tangent like Claes Johnson.
    They persue a theory to the extreme.

    23

  • #
    Greg Goodman

    Talking of hot-spots:

    http://joannenova.com.au/2008/10/the-missing-hotspot/

    Unfortunately the graphs are not directly comparable. One is expressed in deg C change in 44 years, the other is a decadal trend figure. However, roughly converting the data and colour scales it would appear that the “hot-spot” form is similar to that produced by climate models but only about half the amplitude. More or a warm spot than a hot-spot.

    This seems consistent with the mean rate of warming also only being about half what the alarmists predicted.

    It remains to be seen whether this warming is due to GHG or to changes in stratospheric chemistry resulting from the two major eruptions of the late 20th. c.

    15

  • #
    Gee Aye

    Stephen needs to publish this asap to save all those people wasting their time researching blind alleys and our money. It is his moral obligation.

    68

    • #
      AndyG55

      He just has.

      Jo Nova gets far more viewings than many journals..

      97

      • #

        Oh really. I mean he needs to remove the fluff and put in some science so that it is of a standard that will be published in places that don’t publish any old things and which are therefore generally ignored.

        06

        • #

          All I have done is apply well known physics to the issue of surface warming.

          It is the application to AGW theory that is new, not the underlying science. This is a matter which should have been obvious all along and does not require complex justification or quantification.

          I’m quite sure that unless someone with political power takes this and runs with it then it will be ignored but here it remains for all to see and consider.

          911

          • #

            Stephen… people with political power have standards too. Unless they are stupid or corrupt, they want to take action on independent verifiable evidence or listen to people they trust who have sifted through the literature and drawn a conclusion from such evidence. Posting here and in this way is a waste of your time if your goal is to make change. If your goal is to impress your friends and make some more friends you have done fine.

            00

            • #

              Gee Aye,

              My goal is neither to make change nor make friends.

              My goal is to find the truth.

              If the truth is found then let the cards lie where they fall.

              70

              • #
                LtCusper

                “My goal is to find the truth.”

                You have far to go to reach your goal. In order to arrive at the truth Stephen, you will need to show your analysis agrees with proper test of nature. For example, your analysis blue line in Fig. 4 needs to have some points plotted on it from say relevant radiosondes (ref.d) showing reasonable agreement with actual nature. The break point along the blue line being observed in nature’s data would show your analysis has some truth, otherwise your analysis is just another opinion on the internet.

                The same for the green line in that Fig. 4., reasonable radiosonde or similar test agreement is needed to show that is the truth. There is a huge public database of radiosondes from the tropics to the poles gathered at huge expense and usefully employed by researchers in the field that reasonably arrive at nature’s truth.

                Your red line in Fig. 4 needs to also be supported by test data. To find the truth, you will need to dig into a lot of existing research data. None of which you show having been accomplished in this thread.

                Critical Note 1: I do not understand your “hydrostatic balance” horizontal dotted line in Fig.s 2,3,4. Your linked source “Vertical structure of the atmosphere – Hydrostatic balance” shows the hydrostatic eqn. on slide 7: dp/dz as a function of z, not a constant z height. I am guessing what you mean is “tropopause” (where the fluid ceases to be warmed from below & is warmed from above thereby reducing out convection) for that horizontal line. If so, there should be a finite z distance where T is constant above that line as, for example, the 8-9km in the standard midlatitude atmosphere committee vote for their radiative convective conditions observed by radiosonde et. al.

                Critical Note 2: Your graph “Cooling slowed by radiation from surface” would be improved to instead show MALR (slower –5 to -6.5 K/km Frank abbrev.) as opposed to your DALR -9.8 K/km dotted line.

                Critical Note 3: Your arrow head on the abscissa should go to the left indicating T increases to the left, add in Kelvin units too, with km units on the ordinate showing the “big picture”.

                Critical Note 4: Red verbiage in Fig. 4, CO2 has not condensed out of the red line hence you ignore CO2 effect all the while seeking to find CO2 (i.e. IR active gas) effect. Test data will show red line is not parallel to DALR in nature. Prove that statement wrong by actually researching the test data Stephen (or provide an accomplished cite referencing the test data), not just by prose.

                143

              • #

                LtCusper:

                I think it is pretty incontrovertible that water vapour absorbs most at low levels and radiates most at higher levels from the condensate so there is no need to plot points along the blue line in Fig 4. Such points would vary hugely because of the different heights at which the vapour condenses out in different locations. Do you have any radiosonde data that suggests something different ?

                For the green line in Fig 4 I did provide evidence in the cloud tops and tropopause diagram which clearly shows the height discontinuities between rising and falling columns.

                The red line in Fig 4 represents the DALR because the descending air is dry. Are you suggesting that dry descending air does not warm at the DALR?

                CN1 – The height of hydrostatic balance is highly variable within updraughts and downdraughts so it is reasonable to indicate an averge height for present purposes. It is generally around the centre of mass of the atmosphere and so not at the tropopause.

                CN2 – The diagram is sufficient to show the point that absorption causes the environmental lapse rate to diverge to the warm side of the DALR.

                CN3 – There is an arrow pointing to the left.

                CN4 – Fig 4 relates only to water vapour, no CO2. See Fig 3 for CO2 where I have shown the CO2 effect.

                Have you really read it at all?

                79

              • #
                LtCusper

                Stephen – If you want to persuade others as seems to be your intent, natural data is required or you will fail. If the data is incontrovertible as you write, the data should be easy to obtain & use for that purpose. Get the data to find your truth, that endeavor is not the burden of the reader.

                Yes, I read it all, not persuaded without data & two arrows in different directions make it hard to read. You do not make it clear no CO2 in Fig. 4 which makes data impossible to obtain. Thus you will fail to persuade the audience you seek to influence (informed, critical readers).

                132

              • #

                My intent is to find the truth and not necessarily to persuade others though one would hope that to be a side effect.

                I weigh the responses carefully but as yet there seems to be nothing fatal from informed, critical readers and I have quite a few informed critical types on board in a variety of blogs.

                No work satisfies everyone even when it is correct.

                710

              • #
                LtCusper

                Stephen – As I wrote, you cannot get to the truth without proper supporting natural data. Without that data, you have not shown your views are correct. You simply have an opinion.

                112

    • #
      Greg Goodman

      I don’t see anything radically new here, that merits publishing.

      Increased convection is what was supposed to cause the upper tropo tropical hot-spot. This is basic climatology, not new science.

      I don’t see any reply from SW on that point.

      Such a hot-spot exists but is more of a warm-spot. As I pointed out above with graphs from Jo’s page about the ‘hot-spot’, it seems that the warming was about half that predicted by models.

      This is observational evidence that the hypothesised, positive water vapour feedback that was *assumed* to double the scientifically calculated CO2 forcing does not happen. It if a falsified hypothesis.

      68

      • #

        Greg,

        Where is there a hot spot in my diagrams?

        There is a warm spot at the inflection point in the ascending column and a cool spot at the inflection point in the descending column and I accept that due to the different heights at which water vapour may condense out the warm spot in the ascending column would be rather more variable in height than I have indicated.

        I am giving the simplified big picture rather than tackling every detail.

        My proposition is indeed radically new in the context of the current climate debate but it simply reflects an application of known science which should have been obvious all along.

        Furthermore it is consistent with David’s new model architecture and by way of the convective adjustments arising from the lapse rate slope changes provides the mechanism whereby outgoing radiation can be shifted between the various available pipes as necessary.

        This matter needs serious consideration by those in authority and cannot be ignored forever now that it is in the public domain.

        Nobody here has yet put forward anything that could invalidate the basic proposition that lapse rate and convection changes can neutralise any net thermal effect from radiative gases.

        1018

        • #

          Red thumbs, but no rational exposition ?

          90

          • #
            Roy Hogue

            It must be the CO2, Stephen. What else could account for such irrational behavior? It has exceeded someone’s safe limit so rationality isn’t possible anymore. Warmists are very sensitive you know. 😉

            96

        • #

          Stephen Wilde October 25, 2015 at 7:16 pm

          “Nobody here has yet put forward anything that could invalidate the basic proposition that lapse rate and convection changes can neutralise any net thermal effect from radiative gases.”

          Nobody here except you claims the existance any of your “net thermal effect from radiative gases”, or anything else that should be neutralized! Where is the evidence of such?

          110

  • #
    Don Gaddes

    Extract from ‘Tomorrow’s Weather’ Alex S. Gaddes (1990)
    ” However, (see Ref. Nos. 12 and 13) it [rising carbon dioxide level] could well turn out to be lucky for us, in the light of those other factors which act in the opposite way to carbon dioxide in the atmosphere. I refer to wind-blown and volcanic dust in the atmosphere and the ever-intensifying albedo factor.

    May be glad of Greenhouse Effect

    Who can say that sometime in the not-too-distant future, we will not be glad of the “extra blanket” provided by an enhanced “Greenhouse Effect?”

    Apart from the anti-Greenhouse Effect factors mentioned above, there is another powerfully active component at work, which governs the amount of CO2 in the atmosphere.

    According to Rankama and Sahama (Ref. No. 14) the oceans act as a sink for any build-up in carbon dioxide above a partial pressure (PP) in the atmosphere; that once the critical PP in the atmosphere is reached, the excess carbon dioxide is taken into solution by the oceans.

    They note that the cold polar waters are capable of the greatest absorption of carbon dioxide. Apparently the equilibrium ratio of carbon dioxide in sea water to that in the atmosphere is 50:1…..”
    This ‘Carbon Sink’ capacity of the oceans continues to be ignored by David Evans – and, (of course) Steven Wilde.
    The importance of Convection will become more apparent in future discussion on the extrapolation of the ‘X Factor’ to the surface of the Sun.
    Evans’ so called ‘notch’ would (rather) seem to be a manifestation of convection that carries the ‘X Factor’ to the Sunspot latitudes from within the Sun.
    Further extract from ‘Tomorrow’s Weather’ Alex S. Gaddes (1990)

    “Unknown Entity

    (a) It appears to me that the unknown entity, which ends up as a visible sunspot, is carried from its place of origin by a convection cell. Granting this and that, there would also be the possibility of a cell being retarded, unduly, in its journey out to the surface of the Sun, by competition with other convection cells, or other factors. The erratic nature of the controversial ’11 year’ sunspot ‘cycle’ could thus be explained.
    (b) The inference drawn from this quotation is that there are, indeed, two (or more) processes going on in the Sun, which conjointly, appear to be responsible for solar magnetic phenomena and sunspots as well.

    There is also a strong indication that the “double-humps” that he (G. M. Brown) talks about in both the AQD and the sunspot cycles, are likely to turn out to represent harmonic ‘beats’ between different cycle trains. I have a strong feeling that the further we proceed with this study, the more synonymous the ‘beat’ will become with the climate/weather cycle…..”

    An updated version of ‘Tomorrow’s Weather’ Alex S. Gaddes (1990)- including ‘Dry Cycle’ forecasts to 2055, is available as a free pdf from [email protected]

    02

  • #

    I can summarise the most important point of my article as follows:

    At the tropopause, the faster convection in the ascending column in the upper part of the atmosphere pushes up into the stratosphere and then the colder higher air is transferred laterally and downward to entrain colder air into the entire descent column so as to distort the lapse rate to the cold side and offset the distortion to the warm side in the ascending column.

    One cannot have a height change at the surface so events there unfold differently.

    Absorption by CO2 in the 15u wavelength below the CO2 emissions level slows down convection in the lower atmosphere.

    That is, it slows both upward convection in the ascent column AND downward convection in the descent column.

    Warming in the lower part of the descent reduces air density which works to slow the speed of descent towards the surface because colder denser air descends faster and warmer less dense air descends more slowly.

    Warming in the ascent column below the CO2 emissions level also reduces air density but that reduces the lapse rate slope away from the surface which slows convection from the surface.

    So, not only is there less decompression in the ascending column (leading to surface warming) but also there is less compression in the descending column (leading to surface cooling).

    Due to the vertical displacement between the tops of the two columns the descent column then fails to be compressed enough in the lower troposphere to completely negate the entrained colder air from above.

    The surface below the ascending column warms up a little more than the DALR would suggest but the surface below the descending column warms up a little less than the DALR would suggest.

    Horizontal winds at the surface flowing between warm air in low pressure cells and cold air in high pressure cells then equalise the surface variations back to the DALR temperature leaving the extra energy from the CO2 stored in that extra potential energy above the tropopause in pockets above ascending columns as per my article.

    CO2 both slows decompression in uplift for a warming effect AND slows compression in descent for a cooling effect The two cancel out leaving the surface temperature unchanged and the ‘surplus’ energy from the CO2 stored high up in the troposphere.

    Any PE reservoir held in pockets high up in the troposphere is then added to or drawn upon as necessary in response to ANY radiative imbalances from any cause. The process of adding to it is effected by more vigorous convection in the upper portion of the tropopause (more GHGs) and the process of drawing upon it is effected by less vigorous convection in the upper portion of the tropopause (less GHGs).

    Although I refer to GHGs as a relevant variable the same effect occurs from any net changes in the overall radiative interaction between all the various pipes.
    If all the pipes fail to achieve immediate radiative balance by redistributing the radiative energy flows between each other, the PE storage mechanism that I have just described mops up any imbalances until convection adjusts as necessary. It acts as a buffer which gives time for convective adjustments to do their work. In practice, there are barely perceptible changes in the size or vigour of convective cells around the world in response to radiative imbalances.

    713

  • #
    4TimesAYear

    “10” Stars (Dang shakey fingers!)

    30

  • #

    “a more stable lower troposphere with less overturning, as appears to be the case according to the better radiosonde data from 1973.”

    That is exactly what we would expect to see if absorption of energy by CO2 in the lower tropopause reduces the steepness of the lapse rate slope.

    Such a reduction in steepness would result in less vigorous convection so that water vapour condensate does not rise as high in the atmosphere and so radiates to space from a lower, warmer location.

    As CO2 distorts the lapse rate slope in rising columns of air there is a concurrent convective adjustment which affects the water vapour emissions pipe so that correspondingly more radiation escapes to space from the water vapour emissions level.

    2111

    • #
      ianl8888

      … absorption of energy by CO2 in the lower tropopause reduces the steepness of the lapse rate slope.
      Such a reduction in steepness would result in less vigorous convection so that water vapour condensate does not rise as high in the atmosphere and so radiates to space from a lower, warmer location

      That I can see …

      As CO2 distorts the lapse rate slope in rising columns of air there is a concurrent convective adjustment which affects the water vapour emissions pipe …

      But why should there be a “concurrent convective adjustment” ?

      A non-sequitur to me at this point

      64

      • #
        Bobl

        What I think he is saying is that CO2 raises the efficiency of cooling so that for a given amount of convection you get much more radiation to space. Thus the same amount of heat can be moved to space with less Convective overturning, or alternatively for the same amount of convective overturning more radiation occurs .

        Stephen, it also occurs to me that more water content will cause a greater decrease in air pressure when the water condenses out INCREASING convection. The lower lapse rate is a result of more energy moving up the air column, not less. I can’t see how the adjustment to that can reduce convection overall?

        34

        • #

          Bobl,

          I am saying that less powerful convection will lead to a build up of more humidity near the surface and allow clouds at a warmer lower level that radiate more energy out to space than they would at a higher colder level.

          Water vapour condenses out at a higher temperature when humidity is higher.

          “Dew point is a measure of atmospheric moisture. It is the temperature to which air must be cooled to reach saturation (assuming air pressure and moisture content are constant). A higher dew point indicates more moisture present in the air”

          https://en.wikipedia.org/wiki/Dew_point

          Clouds form at a lower warmer height when humidity rises.

          The less steep lapse rate is caused by radiative absorption of IR by CO2 preventing the air from cooling at the rate that would otherwise be set by the dry adiabatic lapse rate.

          Warmer air aloft reduces the speed of upward convection from the surface below by reducing the ambient lapse rate slope.

          It is true that when water vapour condenses out then it does release latent heat which then increases the lapse rate slope between the newly formed cloud and the higher levels so that convection temporarily increases locally but here we are considering global averages and not local disturbances in the averages.

          If absorption by CO2 in the lower troposphere reduces the rate of cooling with height to less than the dry adiabatic lapse rate then convection will slow down, humidity at lower levels will increase, humidity at higher leve;ls will decline, the WVEL will descend, clouds will form at a lower warmer height so that they radiate more effectively to space and there will be no hot spot as envisaged by AGW theory. exactly as observed by the radiosonde data.

          The reduced rate of cooling with height is not a result of more energy moving up the air column. However hot the surface becomes the rate of cooling with height via the dry adiabatic lapse rate is set by mass and gravity creating a density gradient with height and remains the same regardless of the amount of energy moving up the column.

          When condensation releaes latent heat then that latent heat disappears with height gained according to the dry adiabatic lapse rate however much energy is released at any given moment.

          129

          • #

            “Clouds form at a lower warmer height when humidity rises.”

            Why is humidity rising? Relative or specific? Does increasing atmospheric CO2 level do this everywhere?

            “The less steep lapse rate is caused by radiative absorption of IR by CO2 preventing the air from cooling at the rate that would otherwise be set by the dry adiabatic lapse rate.”

            Can you please show any change in lapse rate from any change in atmospheric CO2?

            “Warmer air aloft reduces the speed of upward convection from the surface below by reducing the ambient lapse rate slope.”

            Again a claimed reduction in speed of upward convection with no meaning and no evidence whatsoever!

            “It is true that when water vapour condenses out then it does release latent heat which then increases the lapse rate slope between the newly formed cloud and the higher levels so that convection temporarily increases locally but here we are considering global averages and not local disturbances in the averages.”

            And you evidence of such fantasy claim is where?

            “If absorption by CO2 in the lower troposphere reduces the rate of cooling with height to less than the dry adiabatic lapse rate”

            Temperature lapse must remain independent of any CO2 effect!

            “then convection will slow down, humidity at lower levels will increase, humidity at higher leve;ls will decline,”

            Why?

            “the WVEL will descend, clouds will form at a lower warmer height so that they radiate more effectively to space and there will be no adiabatic lapse rate then convection will slow down,”

            Why?

            “humidity at lower levels will increase, humidity at higher leve;ls will decline, the WVEL will descend,”

            Why?

            “clouds will form at a lower warmer height so that they radiate more effectively to space”
            Why?

            All fantasy and no evidence that any is true, correct, or even possible!

            “The reduced rate of cooling with height is not a result of more energy moving up the air column. However hot the surface becomes the rate of cooling with height via the dry adiabatic lapse rate is set by mass and gravity creating a density gradient with height and remains the same regardless of the amount of energy moving up the column.”

            This may or may not be true! This has never been verified by anyone!

            “When condensation releaes latent heat then that latent heat disappears with height gained according to the dry adiabatic lapse rate however much energy is released at any given moment.”

            More nonsense When condensation releases latent heat that latent heat mostly descends with the water condensate to the cloud base and often re-evaporates!

            213

        • #

          “Stephen, it also occurs to me that more water content will cause a greater decrease in air pressure when the water condenses out INCREASING convection. The lower lapse rate is a result of more energy moving up the air column, not less. I can’t see how the adjustment to that can reduce convection overall?”

          The reduction in apparent volume can be no more than 4% with all WV condensing at one point. There is no change in pressure or density as the water condensate has not precipitated before gravity can re-establish local self buoyancy at the speed of sound!

          211

      • #

        ianl8888,

        Convection occurs when there is a declining temperature gradient with height because lower warmer air has more buoyancy than the colder air immediately above it. In our atmosphere the reduction in pressure and density with height leads to a fixed rate of decline in temperature with heightknown as the dry adiabatic lapse rate.

        The steeper the reduction of temperature with height the faster will be convection as one can see with the development of convective instability when colder higher air is advected in above warmer lower air.

        The speed of upward convection is directly related to the steepness of the rate of temperature decline with height.

        106

        • #

          Again absolutely no definition for words used! Only lawyerly words!

          What meaning can “The speed of upward convection is directly related to the steepness of the rate of temperature decline with height”? What is speeding?
          Where do you get such fantasy?

          110

          • #

            Will,

            You really must do some reading.

            Maybe start here:

            https://en.wikipedia.org/wiki/Natural_convection

            141

            • #

              Stephen Wilde November 2, 2015 at 7:05 pm ·

              Will,You really must do some reading.
              Maybe start here:
              https://en.wikipedia.org/wiki/Natural_convection

              Gives no information about rates of anything or what may affect such rates, should they exist!
              Where is anything of your claimed atmospheric convection and the rates thereof! 🙁

              19

              • #

                Here you go:

                “Natural convection will be more likely and/or more rapid with a greater variation in density between the two fluids, a larger acceleration due to gravity that drives the convection, and/or a larger distance through the convecting medium”

                Uneven temperature differentials create greater variations in density and thus variable rates of lapse rate slope which then lead to variations in the speed of convection.

                130

              • #

                Copy from #41
                Stephen Wilde November 2, 2015 at 9:28 pm

                Here you go:
                “Natural convection will be more likely and/or more rapid with a greater variation in density between the two fluids, a larger acceleration due to gravity that drives the convection, and/or a larger distance through the convecting medium”

                Pulls something from wiki, does not read but posts anyhow!!!

                What two fluids? Just what is being convected in your reference? Does your reference pertain to an atmosphere? How?

                “Uneven temperature differentials create greater variations in density and thus variable rates of lapse rate slope which then lead to variations in the speed of convection.”

                Just where in this compressible atmosphere with all pressure density and temperature dictated by gravity and only modified by the local variation of WV creation or depletion do you find this variation in density, slope and speed of convection (whatever that may mean)! All just rambling prose with no understanding!!

                13

        • #
          Bobl

          Stephen,

          Maybe some rethinking required!

          Convection occurs when there is a declining PRESSURE gradient such that the force of air pressure upward exceeds the gravitational force downward. movement of air is not caused by heat, it is caused by pressure differences sometimes driven by heat, but mostly by condensation and the rotation and orbit of the earth and moon. Heat will only spontaneously flow from hot to cold by conduction, air pressure, KE of wind, latent heat of evaporation /melting and IR are not heat , although they are exchangeable with heat.

          If the GLOBAL atmosphere is more waterlogged then the condensation of water GLOBALLY must be greater causing more updrafts GLOBALLY. The sum of all those little local effects. However I don’t doubt there is a compensating negative feedback or saturation, in nature negative feedback rules.

          03

          • #

            Well, not by me or the established meteorology 🙂

            Heating alters density. Condensation releases latent heat which reduces density.

            If a CO2 molecle warms by absorption of IR from the surface then it will warm the local gases by conduction and the additional warmth at that height reduces the lapse rate slope and inhibits convection from below.

            It is proved by observation that the faster the decline in temperature with height the more unstable the air is and the more vigorous will be upward convection.

            The slower the decline in temperature with height the more stable the air is and the less vigorous will be upward convection.

            Your error is in thinking in absolutes. Instead it is a matter of relative densities and temperatures between air parcels and their surroundings and not the absolute value of either.

            91

            • #
              bobl

              Um, no, not just because you say so, air movement is caused by pressure differences, if air is heated in a given constrained volume, like at the bottom of the atmosphere, the PRESSURE goes up, PV=nRT. Basic physics. Pressure is proportional to temperature, all other things being equal. Yes, I agree that hot air more sparse and weighs less than cold air, and so floats up above the cold air. But this is just another way of saying the hotter higher pressure at the surface migrates to the cooler lower pressure at height. Same thing.

              You also neglect air pressure variation on condensation PV=nRT, whats n? Its the amount of gas, the number of moles, if I take a parcel of air at just above the dew point of 1 mole that is 10% water at say 1000hPa, then cool it fractionally to the point that the water condenses out, what happens to the pressure, n goes from 1 to 0.9 moles. PV=nRT, V,R and T are nearly constant, so the pressure drops to 900hPa.

              00

              • #

                I neglect neither air pressure nor PV=nRT.

                It just doesn’t work the way you have envisioned it.

                It is a matter of relative densities and temperatures between air parcels and their surroundings and not the absolute value of either.

                There are lots of good sources for you and interested parties to refer to and I don’t intend to rehearse it all here.

                80

              • #

                Bob,
                From #16 comment # 28 Will Janoschka Nov 1, 2015 at 11:44 pm
                Dist to food says the spyder: d ≤ h+w+l ≥ √(h²+w²+l²) Run now, or wait ’till Stephen gets closer? Always questions.

                22.4 liter open container (cup) at 101.6 kPa STP has one mole of atmosphere. How many moles of atmosphere in the 22.4 liter cup at 50 kPa pressure altitude? 25kPa? 12.5kPa? How many know how this atmosphere works? certainly not Stephen! Yum food!!! 🙁
                All the best? -will-

                01

            • #

              Stephen Wilde November 3, 2015 at 9:44 am

              Well, not by me or the established meteorology

              Both need flushing long ago!! 🙂

              “Heating alters density.”

              Not of a gas contained within a volume! Is this how your disgraced meteorology works?

              “Condensation releases latent heat which reduces density.”

              Condensation is a definition of increased density! Is this how your disgraced meteorology works?

              “If a CO2 molecle warms by absorption of IR from the surface then it will warm the local gases by conduction and the additional warmth at that height reduces the lapse rate slope and inhibits convection from below.”

              Lotsa Ifs there! Measured any CO2 molecules warming? Can you show any inhibiting of convection?

              “It is proved by observation that the faster the decline in temperature with height the more unstable the air is and the more vigorous will be upward convection.”

              Wilde Bovine Excrement! It is proven by observation and measurement that if for some reason (overlaying cold advection air mass) if the 9.8°C/km gravitational lapse is exceeded; gravitational force will reduce that by any means necessary, conduction, convection, pressure waves, or downright violence! Only disgraced meteorologists calls this UNSTABLE! Anyone with any sense gets the hell out of the way!

              05

    • #

      “Such a reduction in steepness would result in less vigorous convection so that water vapour condensate does not rise as high in the atmosphere and so radiates to space from a lower, warmer location.”

      Climate troll, can you say ever what you may mean by “less vigorous convection”? Is this a lower amount of atmospheric mass elevated to some altitude? Is this a lower speed/velocity of that mass motion to some altitude? Is this less sensible or latent heat elevated to some altitude? What altitude?

      Right here, right now, from the meteorological likes of Stephen and the rest of the CCC, the bovine fecal matter hits the fan!

      What have you ever measured to make such a claim? You have no answers, as there in no defined mechanism to promote your fantasy of hydrostatic equilibrium, air parcel, adiabatic vertical motion, or “work required” for such mass movement!

      Since you know all about this atmosphere and how it works Please answer the following as a repeat:
      From #16 comment # 28 Will Janoschka Nov 1, 2015 at 11:44 pm
      Dist to food says the spyder: d ≤ h+w+l ≥ √(h²+w²+l²)

      22.4 liter open container (cup) at 101.6 kPa STP has one mole of atmosphere. How many moles of atmosphere in the 22.4 liter cup at 50 kPa pressure altitude? 25kPa? 12.5kPa? How many know how this atmosphere works? certainly not Stephen! Yum food!!! 🙁
      All the best? -will-

      414

      • #
        Bobl

        Will
        Please try to be courteous when pointing out others inaccuracies.

        For the record, as I see it, to lower the lapse rate, the flow of energy from the surface to any height has to be greater, this can’t be accomplished with less convection

        It is just possible though if the energy is carried as latent heat and released lower, but condensing out more water lower would cause a rapid pressure drop, a greater pressure gradient and more low level cloud formation and stronger updrafts (due to the greater pressure gradient). In either scenario I can’t see convection being lower.

        Stephen?

        32

        • #

          Bobl November 2, 2015 at 6:03 pm

          “Will
          Please try to be courteous when pointing out others inaccuracies.”

          OK, But not to academic establishments. Bob, this has nothing to do with Stephen, except he actively promotes the fantasies of disgraced meteorology. They obviously have no science, yet is the only group responsible for the CAGW scam! — They, after 200 years using fantasy that would embarrass an astrologist, to only scam others, got well scammed by the AlGorestas, who used but the very same fantasy trick on meteorologists.
          The results are now history. There is still much work to be done flushing all the rest of meteorology. Then we must start all over again with some that wish to learn “ATMOSPHERE”
          All the best! -will-

          311

          • #
            Peter C

            , this has nothing to do with Stephen, except he actively promotes the fantasies of disgraced meteorology. They obviously have no science, yet is the only group responsible for the CAGW scam!

            Well yes, the World Meteorological Organisation has a lot to answer for when it comes to the GAGW scam. They started the whole IPCC thing.

            However, there is quite a lot of science! Temperature records, Balloon flights into the strosphere, Hadley cells etc.

            I do not see how Stephen Wilde promotes fantasies of disgraced meteorology.

            I would say that he is promoting established views of correct meteorology!

            Please explain your alternative view.

            77

            • #
            • #

              Peter C November 2, 2015 at 7:49 pm · Reply

              ( WJ”this has nothing to do with Stephen, except he actively promotes the fantasies of disgraced meteorology. They obviously have no science, yet is the only group responsible for the CAGW scam!”)

              “Well yes, the World Meteorological Organisation has a lot to answer for when it comes to the GAGW scam. They started the whole IPCC thing.”

              “However, there is quite a lot of science! Temperature records, Balloon flights into the strosphere, Hadley cells etc.”

              I do not fault those that observe, recored, chart, and try to predict mostly cyclic but never repeatable phenomena! The astrologers did that well, especially for crops.
              It is the arrogant academics that claim they know how the atmosphere works, that I despise! These ones that oversee all from NASA and NOAA deliberately ignore all that has been learned of the atmosphere since flight. They but regurgitate nonsense like adiabatic air parcels that only can be used to destroy learning. There is nothing about EMR and how it may work in an atmosphere.

              There really is no scientific dicipline there at all!

              “I do not see how Stephen Wilde promotes fantasies of disgraced meteorology. I would say that he is promoting established views of correct meteorology!”

              Stephens writings are always more bizarre than what you call correct meteorology There is never any substance as he simply does not have even that much education or experience! He reads the latest things, throws a way all intended meaning and instead tells a colorful story that some believe. This is indeed the established views of the classical meteorological scam!

              “Please explain your alternative view.”

              Believe nothing! Re-examin everything! Go to the aircraft engineers that get things correct every once in a while. Do not go to a lawyer!

              25

    • #

      “That is exactly what we would expect to see if absorption of energy by CO2 in the lower tropopause reduces the steepness of the lapse rate slope.”

      How is energy absorbed by CO2 in the lower tropopause? Energy is released to space from the tropopause CO2, with the energy of the tropical tropopause supplied by convection, never by EMR flux to!!! Do you ever read what you spout? A reduced magnitude of temperature lapse in the lower troposphere is why the higher local surface temperature develops. Like all the CCC all you ever present is upside down, backward, or both!! 🙁

      No more at #1 OK. Guys. OT. Move this discussion to Stephens thread. Thanks.

      19

  • #

    As per the request of our gracious hostess!

    Peter C November 2, 2015 at 7:49 pm · Reply

    ( WJ”this has nothing to do with Stephen, except he actively promotes the fantasies of disgraced meteorology. They obviously have no science, yet is the only group responsible for the CAGW scam!”)

    “Well yes, the World Meteorological Organisation has a lot to answer for when it comes to the GAGW scam. They started the whole IPCC thing.”

    “However, there is quite a lot of science! Temperature records, Balloon flights into the strosphere, Hadley cells etc.”

    I do not fault those that observe, recored, chart, and try to predict mostly cyclic but never repeatable phenomena! The astrologers did that well, especially for crops.
    It is the arrogant academics that claim they know how the atmosphere works, that I despise! These ones that oversee all from NASA and NOAA deliberately ignore all that has been learned of the atmosphere since flight. They but regurgitate nonsense like adiabatic air parcels that only can be used to destroy learning. There is nothing about EMR and how it may work in an atmosphere.
    There really is no scientific dicipline there at all!

    “I do not see how Stephen Wilde promotes fantasies of disgraced meteorology. I would say that he is promoting established views of correct meteorology!”

    Stephens writings are always more bizarre than what you call correct meteorology There is never any substance as he simply does not have even that much education or experience! He reads the latest things, throws a way all intended meaning and instead tells a colorful story that some believe. This is indeed the established views of the classical meteorological scam!

    “Please explain your alternative view.”

    Believe nothing! Re-examin everything! Go to the aircraft engineers that get things correct every once in a while. Do not go to a lawyer!

    010

  • #

    Will Janoschka November 2, 2015 at 7:28 pm
    Stephen Wilde November 2, 2015 at 7:05 pm

    ((‘Will,You really must do some reading.
    Maybe start here:
    https://en.wikipedia.org/wiki/Natural_convection ‘))

    (“Gives no information about rates of anything or what may affect such rates, should they exist! Where is anything of your claimed atmospheric convection and the rates thereof! 🙁 “)

    Stephen Wilde November 2, 2015 at 9:28 pm #17

    Here you go:
    “Natural convection will be more likely and/or more rapid with a greater variation in density between the two fluids, a larger acceleration due to gravity that drives the convection, and/or a larger distance through the convecting medium”

    Pulls something from wiki, does not read but posts anyhow!!!
    What two fluids? Just what is being convected in your reference? Does your reference pertain to an atmosphere? How?

    “Uneven temperature differentials create greater variations in density and thus variable rates of lapse rate slope which then lead to variations in the speed of convection.”

    Just where in this compressible atmosphere with all pressure density and temperature dictated by gravity and only modified by the local variation of WV creation or depletion do you find this variation in density, slope and speed of convection (whatever that may mean)! All just rambling prose with no understanding!!
    You have been politely asked to post no more under David #17 comment #1 Please comply!!

    09